Download Semi- Fowler`s

Document related concepts

Prenatal testing wikipedia , lookup

Medical ethics wikipedia , lookup

Dysprosody wikipedia , lookup

Adherence (medicine) wikipedia , lookup

Patient safety wikipedia , lookup

Licensed practical nurse wikipedia , lookup

Electronic prescribing wikipedia , lookup

Nurse anesthetist wikipedia , lookup

List of medical mnemonics wikipedia , lookup

Nurse–client relationship wikipedia , lookup

Transcript
1- Which of the following anatomical terms best describes the motion in the
picture above:
a- Adduction
b- Abduction
c- Supination
d- Pronation
2- Which of the above locations represents the best sites to inspect chest
retractions in a child with lower respiratory tract disorder:
a- A&B
b- B&C
c- C&D
1
d- D&E
3- Your patient's finger stick blood sugar at 07:30 is 300 mg/dl, according to
the following sliding scale; how many unites you would give your patient:
a- 6 i.u
b- 8 i.u
c- 14 i.u
d- 18 i.u
4- The nurse is caring for a diabetic child whose 11 a.m. blood glucose
monitoring check is 269 mg/dL. The physician orders the following coverage
schedule:
150 to 200 mg/dL-2 units of Humulin R
201 to 250 mg/dL-4 units of Humulin R
251 to 300 mg/dL-6 units of Humulin R
301 to 350 mg/dL-8 units of Humulin R
351 to 399 mg/dL-I0 units of Humulin R
Over 400 mg/dL-call the physician
According to the coverage schedule; how many unites you would give your
patient:
a- 4 units
b- 6 units
2
c- 8 units
d- 10 units
5- The following syringe contains morphine sulphate in a concentration of 2
mg/cc, how much total morphine this syringe contains:
a- 0.5 mg
b- I mg
c- 2 mg
d- 4 mg
6- The following syringe contains 2 gm of morphine sulphate, how many
grams are there in 1 ml:
a- 0.5 gm
b- 1 gm
c- 2 gm
d- 4 gm
7- The following condition may develop in
females due to an increase in the production of
which of the following hormones:
a- Androgens
b- Progesterone
c- Estrogen
d- Growth
8- What is the name of the process of movement of a dissolved substance
(solutes) from an area of high concentration to an area of low concentration:
3
a- Effusion
b- Diffusion
c- Emulsion
d- Excretion
4
9- Which of the following is the incision site for an open cholecystectomy
operation:
a- 1
b- 2
c- 3
d- 4
10- Which of the following is the incision site for an open gastrectomy
operation:
a- 1
b- 2
c- 3
d- 4
11- Which of the following is the incision site for surgical appendectomy
operation:
a- A
b- B
c- C
d- D
5
6
12- What is the most probable type of arrhythmia in the following ECG (lead
II):
a- Atrial fibrillation
b- Atrial flutter
c- Ventricular fibrillation
d- Supra-ventricular fibrillation
13- What is the most probable type of arrhythmia in the following ECG (lead
VI):
a- Atrial fibrillation
b- Atrial flutter
c- Ventricular tachycardia
d- Supra-ventricular fibrillation
14- The following picture suggests that the patient has:
a- Renal disease
b- Hepatic disease
c- Pulmonary disease
d- Cerebral disease
15- A patient with colostomy, what type of stool you should suspect:
a- With a sigmoid colostomy, the feces are formed
b- With a descending colostomy, the feces are
semiformed
c- With a transverse colostomy, the feces are
unformed
7
d- With an ascending colostomy, the feces are fluid
8
16- When a patient is vomiting postoperatively, which of the following
positions would the nurse use to protect the patient?
a-
c-
b-
d-
17 - The following picture represents the pain distribution of which nerve
affection:
a- C2
b- C6
c- T3
d- T6
18- On assessing the client's pressure ulcer (see photo), the nurse would
document this as which stage:
a- Stage I
b- Stage II
c- Stage III
d- Stage IV
9
10
19- The following picture represents an auto immune disease known as:
a- Psoriasis
b- Systemic Lupus Erythematosus
c- Molluscum Contagiosum
d- Eczema
20- What is the name of the equipment this picture shows?
a- Oropharyngeal airway (OPA)
b- Nasopharyngeal airway (NPA)
c- Laryngeal mask airway (LMA)
d- Urinary catheter
21- A client is being prepared for a thoracentesis. A nurse assists the client to
which position for the procedure:
a- Lying in bed on the affected side
b- Lying in bed on the unaffected side
c- Sims' position with the head of the bed flat
d- Prone with the head turned to the side and supported by a pillow
11
22- A nurse is preparing to insert a nasogastric tube into a client. The nurse
places the client in which position for insertion:
a- Right side
b- Low Fowler's
c- High Fowler's
d- Supine with the head flat
23- A nurse develops a plan of care for a client with deep vein thrombosis.
Which client position or activity in the plan will be included:
a- Out-of-bed activities as desired
b- Bedrest with the affected extremity kept flat
c- Bedrest with elevation of the affected extremity
d- Bedrest with the affected extremity in a dependent position
24- A client in labor is transported to the delivery room and is prepared for a
cesarean delivery. The client is transferred to the delivery room table, and the
nurse places the client in the:
a- Trendelenburg's position with the legs in stirrups
b- Semi-Fowler position with a pillow under the knees
c- Prone position with the legs separated and elevated
d- Supine position with a wedge under the right hip
25- The nurse is caring for a client who is 1 day postoperative for a total hip
replacement. Which is the best position in which the nurse should place the
client:
a- Side-lying on the operative side
b- On the nonoperative side with the legs abducted
12
c- Side-lying with the affected leg internally rotated
d- Side-lying with the affected leg externally rotated
26- Which of the following is the best position to increase the brain perfusion
(cerebral perfusion):
a- Prone
b- Supine
c- Semi- Fowler's
d- Trendelenburg's
27- A nurse is administering a cleansing enema to a client with a fecal
impaction. Before administering the enema, the nurse places the client in
which position:
a- Left Sims' position
b- Right Sims' position
c- On the left side of the body, with the head of the bed elevated 45
degrees
d- On the right side of the body, with the head of the bed elevated 45
degrees
28- A client has just returned to a nursing unit after an above-knee
amputation of the right leg. A nurse places the client in which position:
a- Prone
b- Reverse Trendelenburg's
c- Supine, with the amputated limb flat on the bed
d- Supine, with the amputated limb supported with pillows
29- A nurse is caring for a client with a severe burn who is scheduled for an
autograft to be placed on the lower extremity. The nurse develops a
13
postoperative plan of care for the client and includes which of the following
in the plan:
a- Maintain the client in a prone position
b- Elevate and immobilize the grafted extremity
c- Maintain the surgical extremity in a flat position
d- Keep the surgical extremity covered with a blanket
30- A nurse is preparing to care for a client who has returned to the nursing
unit following cardiac catheterization performed through the femoral artery.
The nurse checks the physician's prescription and plans to allow which client
position or activity following the procedure:
a- Bedrest in high Fowler's position
b- Bedrest with bathroom privileges only
c- Bedrest with head elevation at 60 degrees
d- Bedrest with head elevation no greater than 30 degrees
31- After the client undergoes a total hip replacement, how should the nurse
position the affected hip?
a- Adduct the hip
b- Abduct the hip
c- Flex the hip
d- Extend the hip
32- A client has an open reduction and internal fixation for a fractured hip.
Postoperatively the nurse should position the client's affected extremity in:
a- External rotation
b- Slight hip flexion
c- Moderate abduction
14
d- Anatomical body alignment
33- Which of the following positions should the nurse place a client for rectal
tube insertion:
a- left lateral position with hyperextension of right knee
b- left lateral position with hyperflextion of left knee
c- left lateral position with hyperflextion of right knee
d- left
lateral
position
with
15
hyperextension
of
left
knee
34- Which of the following position is appropriate for the patient to use for
selfadministration of fleet enema:
a- Left lateral position right leg flex
b- Face-down and leg flex
c- Face upward and leg free (on back
and flex hip and knee)
d- Chest- knee position
35- The best position for a child with myelomeningocele is?
a- Prone
b- supine
c- semi-fowler's
d- modified trendelenburg
36- Unless the physician orders otherwise, in which position should the nurse
place the infant during the postoperative period of surgical repair of the
myelomeningocele:
a- Supine
b- Prone
c- Right or left side-lying
d- Whichever position is most comfortable for the infant
37- The nurse is caring for a client with an above-the-knee amputation (AKA).
To prevent contractures, the nurse should:
a- Place the client in a prone position 15-30 minutes twice a day
b- Keep the foot of the bed elevated on shock blocks
c- Place trochanter rolls on either side of the affected leg
16
d- Keep the client's leg elevated on two pillows
38- Postoperative nursing care of the infant following surgical repair of a cleft
lip would include:
a- Feeding the infant with a spoon to avoid sucking
b- Positioning the infant on the abdomen to facilitate drainage
c- Applying elbow restraints to protect the surgical area
d- Providing minimal stimulation to prevent injury to the incision
39- The client has returned to the nursing unit following a right below-theknee amputation. How should the nurse position the client?
a- Supine with head turned to the side
b- With shock blocks placed under the foot of the bed
c- Semi-sitting position with knees bent
d- Left lateral with pillows between the knees
40- What position should the nurse place the head of the bed in to obtain the
most accurate reading of jugular vein distention:
a- High fowler's
b- Raised 10 degrees
c- Raised 30 degrees
d- Supine position
41- Which of the following body positions is best for a patient with increased
ICP:
17
a- Prone
b- Trendelenburg
c- HOB elevated 90 degrees and hips flexed
d- HOB elevated 20 degrees
42- Immediately after a percutaneous liver biopsy, the nurse should place the
client in which of the following positions:
a- Left- sided lying
b- Right- sided lying
c- Lithotomy
d- Trendelenburg
43- An adult has low back pain. Which position is likely to be most
comfortable for the client:
a- Prone
b- Supine
c- Side-lying with knees flexed
d- Semi-sitting with legs extended
44- When planning the client's postoperative care, which is the least desirable
position in which the nurse can place the client:
a- Lying prone
b- Lying supine
c- Sitting in a chair
d- Standing to shower
45- Which of the following is the appropriate position for a patient who is
suffering from acute asthma attack is:
18
a- Right lateral
b- Supine
c- High fowler
d- Prone
46- A patient with COPD is admitted to the hospital. How can the nurse best
position the patient to improve gas exchange:
a- Sitting up at the bedside in a chair and leaning slightly forward
b- Resting in bed with the head elevated to 45 to 60 degrees
c- In the Trendelenburg's position with several pillows behind the head
d- Resting in bed in a high-Fowler's position with the knees flexed
47- Following a tympanoplasty, the nurse should maintain the client in which
position:
a- Semi-Fowler's with the operative ear facing down
b- Low Trendelenburg's with the head in neutral position
c- Flat with the head turned to the side with the operative ear facing up
d- Supine with a small neck roll to allow for drainage
48- Which of the following is the proper position for a patient undergoing a
cardiac surgery:
a- Supine
b- Prone
c- Lateral
19
d- Fowler's
49- A nurse is caring for a toddler after surgical repair of a cleft palate. The
nurse should position the child:
a- On his back
b- On his stomach
c- On his back with his head slightly elevated
d- For comfort
50- The nurse observes that the infant's anterior fontanelle is bulging after
placement of a ventriculoperitoneal shunt. The nurse positions this infant:
a- Prone, with the head of the bed elevated
b- Supine, with the head flat
c- Side-lying on the operative side
d- In a semi-fowler's position
51- A nurse is providing care for a client following surgery to remove a
cataract from the right eye. In which position should the nurse place the
client?
a- Right-side lying
b- Prone
c- Supine
d- Trendelenburg's
52- An adult is admitted for a neurological workup and is scheduled for a
spinal tap. When preparing the client for the procedure, the nurse should
position the client in which position:
a- Prone
b- On the side with knees drawn up to chest
20
c- Lithotomy
d- Semi-sitting
53- When suctioning the oropharynx, which of the following is the proper
position of the patient:
a- Prone
b- Supine
c- Semi- Fowler's
d- Trendelenburg's
54- To prevent headache after spinal anesthesia the patient should be
positioned:
a- Semi- fowler's
b- Flat on bed for 6 to 8 hours
c- Prone position
d- Modified Trendelenburg
55- If a patient develops autonomic hyperreflexia the first action his caring
nurse should take is to:
a- Elevate head of bed 90 degree (move from supine to sitting)
b- Make the bed in flat position
c- Apply ice on the axillary and groin
d- But the patient in trendelenburg position
56- To protect susceptible patients in the hospital from aspiration
pneumonia, the nurse will plan to:
a- Turn and reposition immobile patients at least every 2 hours.
b- Position patients with altered consciousness in lateral positions.
c- Monitor frequently for respiratory symptoms in patients who are
immunosuppressed.
21
d- Provide for continuous subglottic aspiration in patients receiving enteral
feedings.
57- Which nursing action is essential when providing continuous enteral
feeding?
a- Elevating the head of the bed
b- Positioning the patient on the left side
c- Warming the formula before administrating it
d- Hanging a full day's worth formula at one time
58- Which of the following positions is appropriate for the patient who has
hypovolemic shock:
a- Prone
b- Supine
c- Semi-fowler's
d- Flat with elevating the feet
59- When caring for a patient admitted post-stroke (CV A) who has altered
consciousness, the nurse should place the patient in which position:
a- Side-lying
b- Supine
c- Prone
d- Semi-fowler's
60- A nurse is providing instructions to a client and the family regarding
home care after right eye cataract removal. Which statement by the client
would indicate an understanding of the instructions:
a- "I should not sleep on my left side."
b- "I should not sleep on my right side. "
c- "I should not sleep with my head elevated."
22
d- "I should not wear my glasses at any time."
61- The day after an amputation, the client begins to hemorrhage from his
stump.
What action should the nurse take first:
a- Apply a pressure dressing to the stump
b- Place a tourniquet above the stump
c- Notify the physician
d- Apply an ice pack to the stump
62- A 72-year-old male client has a total hip replacement for long-standing
degenerative bone disease of the hip. When assessing this client
postoperatively, the nurse considers that the most common complication of
hip surgery is:
a- Pneumonia
b- Hemorrhage
c- Wound infection
d- Pulmonary embolism
63- The nurse is caring for a client who had a right below-the-knee
amputation three days ago. The client complains of pain in the right foot
and asks for pain medication. What nursing action is appropriate initially?
a- Elevate the stump
b- Administer a placebo
c- Administer ordered medications
d- Encourage the client to discuss his feelings
23
64- During the assessment of an injury for a possible fracture, which of the
following manifestations would make you suspicious that a fracture has
occurred:
a- Impaired sensation
b- Hotness
c- Loss of function
d- Pointed tenderness
65- A client who has had an above-the-knee amputation develops a dimesized bright red spot on the dressing after 45 minutes in the postanesthesia
recovery unit. The nurse should:
a- Elevate the stump
b- Reinforce the dressing
c- Call the surgeon
d- Draw a mark around the site
66- How you will interfere to relieve swelling of the lower extremity in a
cast for 3 days now:
a- Change the cast
b- Call the doctor
c- Elevate the extremity
d- Give massage
67- The nurse is developing a bowel-retraining plan for a client with
multiple sclerosis. Which measure is likely to be least helpful to the client:
a- Limiting fluid intake to 1000mL per day
b- Providing a high-roughage diet
c- Elevating the toilet seat for easy access
24
d- Establishing a regular schedule for toileting
68- A diabetic patient with foot gangrene undergone above knee
amputation, while the nurse changing the dressing he complains of pain on
the same knee which was amputated. What should the nurse do:
a- Inform physician about it
b- Re-do dressing to Assessment the wound
c- Psychiatry consultation to the patient because he wound above knee
d- Give analgesic as needed
69- A diabetic patient with foot gangrene undergone above knee
amputation he complain of pain and swelling at the wound site which is
oozing bus and has a bad odor, the physician ordered cephalexin and
metronidazole. Which of the following you should do immediately:
a- Give cephalexin direct first action
b- Give metronidazole direct first action
c- Do wash on wound with N/Sand put bacitracin
d- Do wash on wound with N/Sand put hydrocortisone
70- The physician has prescribed a cleansing enema to a client scheduled
for colon surgery. The nurse would place the client:
a- Prone
b- Supine
c- Left sim's ( left lateral)
d- Dorsal recumbent
71- At which side of the patient will the nurse stand when inserting a rectal
enema:
a- Right side
b- Left side
25
c- Any side
d- Both sides
72- While undergoing a soapsuds enema, the client complains of abdominal
cramping. The nurse should:
a- Immediately stop the infusion
b- Lower the height of the enema container
c- Advance the enema tubing 2 to 3 inches
d- Clamp the tubing
73- The nurse must administer an enema to an adult patient with
constipation. Which of the following would be a safe and effective distance
for the nurse to insert the tubing into the patient's rectum:
a- 1:2 cm
b- 3:4 cm
c- 5.5:6.5 cm
d- 6.5:8 cm
74- The nurse is caring for a patient receiving IV furosemide (Lasix) 40 mg
and Enalapril (Vasotec) 5 mg PO bid for ADHF with severe orthopnea.
When evaluating the patient response to the medications, the best indicator
that the treatment has been effective is:
a- Weight loss of2 pounds overnight.
b- Improvement in hourly urinary output.
c- Reduction in systolic bp.
d- Decreased dyspnea with the head of the bed at 30 degrees.
26
75- The nurse in preparing to insert RYLE'S tube (NGT) into an infant, the
nurse knows that the length of the tube should be taken as following:
a- From the nose down to the chin and then to the umbilicus
b- From the nose to the earlobe and then to the xiphoid process
c- From the nose to the mouth to the xiphoid process
d- From the nose to the earlobe to the umbilicus
76- The nurse is caring for a client who has had a chest tube inserted and
connected to water seal drainage. The nurse determines the drainage
system is functioning correctly when which of the following is observed:
a- Continuous bubbling in the water seal chamber
b- Fluctuation in the water seal chamber
c- Suction tubing attached to a wall unit
d- Vesicular breath sounds throughout the lung fields
77- The nurse is caring for a client who has just had a chest tube attached to
a water seal drainage system. To ensure that the system is functioning
effectively the nurse should:
a- Observe for intermittent bubbling in the water seal chamber
b- Flush the chest tubes with 30-60 ml of NSS every 4-6 hours
c- Maintain the client in an extreme lateral position
d- Strip the chest tubes in the direction of the client
78- The nurse enters the room of a client who has a chest tube attached to a
water seal drainage system and notices the chest tube is dislodged from the
chest. The most appropriate nursing intervention is to:
a- Notify the physician
b- Insert a new chest tube
27
c- Cover the insertion site with petroleum gauze
d- Instruct the client to breathe deeply until help arrives
79- Which type of isolation category is indicated for patient with
tuberculosis:
a- Airborne isolation
b- Strict isolation
c- Reverse isolation
d- Contact isolation
80- While attempting to get out of bed, a patient accidentally disconnects
the chest tube from the Pleur-evac drainage system. Which of the following
actions should the nurse take first?
a- Insert the end of the chest tube in a container of sterile solution
b- Clamp the chest tube near the Pleur -evac drainage system
c- Raise the end of the chest tube above the level of the insertion of the chest
tube
d- Apply pressure dressing to the chest tube insertion site
81- Which type of isolation category is indicated for a burn patient:
a- Airborne isolation
b- Strict isolation
c- Reverse isolation
d- No isolation required
82- Which type of isolation category is indicated for patient with
diphtheria:
28
a- Airborne
b- Droplet
c- Contact
d- Blood
83- A 68-years-old woman diagnosed with thrombocytopenia due to acute
lymphocytic leukemia is admitted to the hospital. The nurse should assign
the patient to a:
a- To a private room so she will not infect other patients and health care
workers.
b- To a private room so she will not be infected by other patients and health
care workers.
c- To a semiprivate room so she will have stimulation during her
hospitalization.
d- To a semiprivate room so she will have the opportunity to express her
feelings about her illness.
84- A nurse who begins to administer medications to a client via a
nasogastric feeding tube suspects that the tube has become clogged. The
nurse should take which safe action first:
a- Aspirate the tube
b- Flush the tube with warm water
c- Prepare to remove and replace the tube
d- Flush with a carbonated liquid such as cola
29
85- Which of the following instructions is appropriate for the nurse to give
to a female client a who complains of abdominal upset after
cholecystectomy operation:
a- Increase fluid intake
b- Avoid fatty meals
c- Increase protein intake
d- Daily exercise
86- Which of the following pulses should be checked before administrating
Digoxin:
a- Apical pulse
b- Radial pulse
c- Femoral pulse
d- Dorsalis pedis pulse
87- Which of the following interventions must the nurse take when
administrating digoxin to the patient:
a- Give him the medication with a glass of orange juice
b- Check him for signs of hypokalemia before giving the medication
c- Instruct him to place the medication under the tongue
d- Withhold the medication if his pulse is less than 60 beats/ minute
88- The nurse must withhold Digoxin from a patient if his pulse rate is:
a- Less than 45/m
b- More than 60/m
c- Less than 60/m
d- More than 100/m
30
89- The nurse is assessing the client for possible evidence of digitalis
toxicity. The nurse acknowledges that which is included in the signs and
symptoms for digitalis toxicity:
a- Pulse (heart) rate of 100 beats/min
b- Pulse of 72 with an irregular rate
c- Pulse greater than 60 beats/min and irregular rate
d- Pulse below 60 beats/min and irregular rate
90- A newly admitted client takes digoxin 0.25 mg/day. The nurse knows
that the serum therapeutic range for digoxin is:
a- 0.1 to 1.5 ng/mL
b- 0.5 to 2.0 ng/mL
c- 1.0 to 2.5 ng/mL
d- 2.0 to 4.0 ng/mL
91- A patient complains of severe pain which he stated to be 9/10, the
physician ordered morphine 50 mg IV every 4 hours, the last dose was
given 2 hours ago, what is the best action his caring nurse would take:
a- Give another dose of morphine
b- Inform the doctor to change the order
c- Distract the patient by TV, radio or games for 2 hours
d- Ignore the patient completely
92- The nurse is caring for a patient admitted 1 week ago with an acute
spinal cord injury. Which of the following assessment findings would alert
the nurse to the presence of autonomic dysreflexia (hyperreflexia)?
a- Tachycardia
b- Hypotension
c- Hot, dry skin
d- Throbbing headache
31
93- Which of the following is the best way for long term feeding for a
patient experiencing severe dysphagia:
a- NGT
b- Naso-dudenal tube
c- Gastrostomy
d- Parenteral
94- Patient records are as following, he had 650cc of IV fluid (n/s), 50 ml of
orange Juice in the NG tube, 100 water with the medications, and urinate
500 cc, and 100 cc collected in the G bag. What is the total intake for this
patient?
a- 800 cc
b- 650 cc
c- 200 cc
d- 600 cc
95- The physician ordered to give the patient 10,000 unit of heparin, the
preparation available is 40,000/ml. how much would the nurse give this
patient:
a- 4 ml
b- 2.5 ml
c- 0.25 ml
d- 25 ml
96- The nurse is caring for a client with a fracture. The client develops a
deep vein thrombosis in the opposite extremity. Physician orders include a
heparin drip of D5W 250 mL with heparin 12,500 units at 16 mL/hr. How
many units of heparin is this client receiving per hour:
a- 200 unit/h
32
b- 400 unit/h
c- 600 unit/h
d- 800 unit/h
97- A patient suffers from diarrhea and muscle weakness, the nurse would
assess this patient for which of the following abnormal laboratory blood
tests:
a- Blood calcium
b- Blood magnesium
c- Blood potassium
d- Blood sodium
100- When assessing for therapeutic effects of mannitol, the nurse would
expect to see:
a- Decreased intracranial pressure
b- Decreased excretion of therapeutic medications
c- Increased urine osmolality
d- Decreased serum osmolality
101- Which drug would be used to treat a patient who has increased
intracranial pressure (ICP) resulting from head trauma after an accident:
a- Mannitol
b- Atropine sulfate
c- Epinephrine hydrochloride
d- Sodium bicarbonate
102- Which of the following is the drug of choice for treatment of cerebral
edema:
33
a- Mannitol
b- Atropine sulfate
c- Epinephrine hydrochloride
d- Sodium bicarbonate
103- The nurse would assess which laboratory value to determine the
effectiveness of intravenous heparin:
a- Complete blood count (CBC)
b- Activated partial thromboplastin time (aPTT)
c- Prothrombin time (PT)
d- BUN
104- The antidote to heparin is:
a- Aspirin
b- Vitamin k
c- Warfarin
d- Protamine sulfate
105- How do we monitor the effectiveness of Coumadin (Warfarin):
a- Monitor vital signs
b- PT/INR
c- Weight
d- Ask the patient how they feel
106- Which medication is used to treat a patient suffering from severe
adverse effects of a narcotic analgesic:
a- Naloxone (Narcan)
34
b- Acetylcysteine (Mucomyst)
c- Methylprednisolone (Solu-Medrol)
d- Protamine sulfate
107-A patient who has been anticoagulated with warfarin (Coumadin) is
admitted with gastrointestinal bleeding. The nurse will anticipate
administering which substance:
a- Vitamin E
b- Vitamin K
c- Protamine sulfate
d- Calcium gluconate
108- While admitting a patient for treatment of an acetaminophen
overdose, the nurse prepares to administer which of the following
medications to prevent toxicity:
a- Naloxone (Narcan)
b- Acetylcysteine (Mucomyst)
c- Methylprednisolone (Solu-Medrol)
d- Vitamin K
109- If the nurse gave a patient antihypertensive drug that is prescribed for
another patient, what should she do:
a- Ignore what she has done
b- Call urgent CPR
c- Document that in nursing report
d- Inform doctor after 2 days
110- When a physician orders an arterial blood gas, which artery is the
appropriate one to use to obtain the specimen:
35
a- Cerebral artery
b- Radial artery
c- Carotid artery
d- Ulnar artery
111- A client has an order to have a set of arterial blood gases (ABG's)
drawn. The intended site is the radial artery. The nurse ensures that which
of the following is positive before the ABGs are drawn
a- Homan's sign
b- Brodzinski's sign
c- Babinski reflex
d- Allen's test
112-A client is scheduled for blood to be drawn from the radial artery for
an ABG determination. A nurse assists with performing Allen's test before
drawing the blood to determine the adequacy of the:
a- Ulnar circulation
b- Carotid circulation
c- Femoral circulation
d- Brachial circulation
113- A nurse has administered approximately half of an enema solution to a
preoperative client when the client complains of pain and cramping. Which
nursing action is the most appropriate:
a- Raise the enema bag so that the solution can be instilled quickly.
b- Clamp the tubing for 30 seconds and restart the flow at a slower rate.
c- Reassure the client and continue the flow.
36
d- Discontinue the enema and notify the physician
114- When caring for a client who is receiving total parenteral nutrition
(TPN), which of the following complications would be most important for
the nurse to assess:
a- Chest pain
b- Hemorrhage and air embolus
c- Pneumonia and hyperglycemia
d- Electrolyte imbalance and sepsis
115- A client has a nasogastric tube after a gastric resection. The nurse
should expect to observe:
a- Vomiting
b- Gastric distention
c- Intermittent periods of diarrhea
d- Bloody drainage for the first 12 hours
116- While assessing a client with hypoparathyroidism, the nurse taps the
client's facial nerve and observes twitching of the mouth and tightening of
the jaw. The nurse would document this finding as which of the following:
a- Positive Trousseau's sign
b- Positive Chvostek's sign
c- Tetany
d- Hyperactive deep tendon reflex
117 - Which of the following clinical manifestations of type 2 diabetes
occurs if glucose levels are very high:
a- Hyperactivity
b- Blurred vision
37
c- Oliguria
d- Increased energy
118- Which of the following would the nurse expect to find in a client with
severe
hyperthyroidism:
a- Tetany
b- Buffalo hump
c- Exophthalmos
d- Striae
119-
A
client
is
receiving
long-term
treatment
with
high-dose
corticosteroids. Which of the following would the nurse expect the client to
exhibit:
a- Weight loss
b- Pale thick skin
c- Hypotension
d- Moon face
120- After undergoing a thyroidectomy, a client develops hypocalcemia and
tetany. Which electrolyte should the nurse anticipate administering:
a- Sodium phosphorus
b- Sodium bicarbonate
c- Calcium gluconate
d- Potassium chloride
121- The nurse who elicits a positive Chvostek's sign would suspect that the
patient has which condition:
a- Hyperkalemia
b- Hypocalcemia
38
c- Hypercalcemia
d- Hypernatremia
122- Which of the following is the best method to decrease confusion and
irritability for asthmatic patient:
a- Give antibiotic
b- Give sedative
c- Give vasodilator
d- Give oxygen
123- Which one of the baseline vital signs that has the most effect on sp02:
a- Heart rate
b- Respiratory rate
c- Blood pressure
d- Temperature
124- A patient is admitted to the emergency department with a stab wound
to the right chest. Air can be heard entering his chest with each inspiration.
To decrease the possibility of a tension pneumothorax in the patient, the
nurse should:
a- Position the patient so that the right chest is dependent.
b- Administer high-flow oxygen using a non-rebreathing mask.
c- Cover the sucking chest wound with an occlusive dressing.
d- Tape a nonporous dressing on three sides over the chest wound.
125- A client with pneumonia has a temperature of 102.60 F (39.2 0 C), is
diaphoretic, and has a productive cough. The nurse should include which of
the following measures in the plan of care:
a- Position changes every 4 hours.
39
b- Nasotracheal suctioning to clear secretions.
c- Frequent linen changes
d- Frequent offering of a bedpan
126- The nurse reviews an arterial blood gas report for a client with chronic
obstructive pulmonary disease (COPD). PH 7.35; PC02 62; P02 70; HC03
34 the nurse should:
a- Apply a 100% non-rebreather mask.
b- Assess the vital signs.
c- Reposition the client.
d- Prepare for intubation
127- If the nurse notes the following symptoms after the client begins taking
sertraline (Zoloft), which one is most likely drug-related:
a- Polyuria
b- Diplopia
c- Drooling
d- Insomnia
128- Which of the following diets would be most appropriate for a client
with chronic obstructive pulmonary disease (COPD):
a- Low-fat, low-cholesterol diet.
b- Bland, soft diet.
c- Low-sodium diet.
d- High-calorie, high-protein diet.
129- The physician has prescribed sertraline (Zoloft) 50 mg daily for a
client with depression. Which finding should be reported to the physician?
a- The client takes Tagamet (cimetidine) for acid reflux
40
b- The client takes the medication with meals
c- The client takes the medication once a day at bedtime
d- The client takes Aleve (naproxen) for arthritis
130- A patient was diagnosed with depression 6 weeks ago, and was
prescribed anti-depressant drug.1 week ago the patient started to complain
of irritability and had difficult sleep for only 3 hours daily. Which of the
following is the best nursing action:
a- Inform the doctor to increase the dose
b- Ignore the patient complaint
c- That's a side effect of the medication
d- Give the patient hypnotics
131- 16 years old girl tried to suicide by taking a large amount of valium
(benzodiazepines), the nurse knows that the antidote for this drug is:
a- Flumazenil
b- Benztropine
c- Meperidine
d- Naloxone
132- The nurse would expect to find which information when reviewing the
history of a client diagnosed with multiple sclerosis:
a- Visual problems
b- Increased sensitivity to pain
c- Ascending weakness and numbness
d- Confusion and disorientation
133- The antidote for morphine overdose is:
a- Naloxone (Narcan)
b- Flumazenil (Romazicon)
41
c- Benztropine (Cogentin)
d- Meperidine (Demerol)
134- The nurse assesses a respiratory rate of 10 on a client with cancer who
has just received a hydromorphone hydrochloride (Dilaudid) injection.
Which drug should the nurse be prepared to administer:
a- Naloxone (Narcan)
b- Flumazenil (Romazicon)
c- Benztropine (Cogentin)
d- Meperidine (Demerol)
135- Which of the following drugs is the antidote for magnesium toxicity:
a- Calcium gluconate (Kalcinate)
b- Hydralazine (Apresoline)
c- Naloxone (Narcan)
d- Rho (D) immune globulin (RhoGAM)
136-A client with myasthenia gravis is suspected of having cholinergic
crisis.
Which of the following indicate that this crisis exists:
a- Ataxia
b- Mouth sores
c- Hypotension
d- Hypertension
137-A client with myasthenia gravis is receiving pyridostigmine (Mestinon).
The nurse monitors for signs and symptoms of cholinergic crisis caused by
overdose of the medication. The nurse checks the medication supply to
ensure that which medication is available for administration if a cholinergic
crisis occurs:
42
a- Vitamin K
b- Atropine sulfate
c- Protamine sulfate
d- Acetylcysteine (Mucomyst)
138- Which of the following is the purpose of providing atropine prior to
anesthesia and surgery:
a- To decrease salivation (decrease secretions)
b- To increase blood pressure
c- To facilitate breathing
d- To decrease muscle tone
139-A client is diagnosed as having secondary Cushing's syndrome. The
nurse knows that the client has most likely been taking which medication:
a- Estrogen
b- Penicillin
c- Lovastatin
d- Prednisone
140- A patient has the following preoperative medication order: morphine
10 mg with atropine 0.4 mg 1M. The nurse informs the patient that this
injection will:
a- Decrease nausea and vomiting during and after surgery
b- Decrease oral and respiratory secretions, thereby drying the mouth
c- Decrease anxiety and produce amnesia of the preoperative period
d- Induce sleep, so the patient will not be aware during transport to the
operating room
141- A 40-year-old woman is admitted in labor with high blood pressure,
edema, and proteinuria. She is started on magnesium sulfate. The nurse
caring for her should be sure to keep which drug at the bedside:
43
a- Calcium gluconate
b- Naloxone (Narcan)
c- Phenytoin (Dilantin)
d- Glucose
142- Which of the following diets would be most appropriate for a patient
has Cushing syndrome:
a- High protein diet
b- High potassium diet
c- High calcium diet
d- High iron diet
143- Vitamin K is prescribed for a neonate. A nurse prepares to administer
the medication in which muscle site:
a- Deltoid
b- Triceps
c- Vastus lateralis
d- Biceps
144- Which of the following interventions should be taken to help an older
client to prevent osteoporosis:
a- Decrease dietary calcium intake
b- Increase sedentary lifestyles
c- Increase dietary protein intake
d- Encourage regular exercise
145-An adult is admitted with heart failure. The nurse notes that he has
neck vein distention and slight peripheral edema. The nurse knows that
these signs indicate which of the following:
a- Pneumothorax
44
b- Right-sided heart failure
c- Cardiogenic shock
d- Left-sided heart failure
146- An adult client is admitted with a diagnosis of left-sided congestive
heart failure. Which assessment finding would most likely be present?
a- Distended neck veins
b- Dyspnea
c- Hepatomegaly
d- Pitting edema
147- When the nurse on duty accidentally bumps the bassinet, the neonate
throws out its arms, hands opened, and begins to cry. The nurse interprets
this reaction as indicative of which of the following reflexes:
a- Moro Reflex
b- Babinski reflex
c- Grasping Reflex
d- Tonic Neck Reflex
148- A patient who has required prolonged mechanical ventilation has the
following arterial blood gas results: pH 7.48, PaO2 85 mm Hg, PaCO2 32
mm Hg, and HCO3 25 mEq/L. The nurse interprets these results as:
a- Metabolic acidosis
b- Metabolic alkalosis
c- Respiratory acidosis
d- Respiratory alkalosis
149- A patient has the following ABG results: pH 7.32, PaO2 88 mm Hg,
PaCO2 37 mm Hg, and HC03 16 mEq/L. The nurse interprets these results
as:
45
a- Respiratory acidosis
b- Respiratory alkalosis
c- Metabolic acidosis
d- Metabolic alkalosis
150- The nurse forgot to give the patient his prescribed drug, when the head
nurse asked who is responsible for this the nurse replayed that was my
wrong this considered:
a- Responsibility
b- Ignorance
c- Abandonment
d- Honor
151- When the patient vomits postoperatively, the most important nursing
objective is to prevent:
a- Dehydration
b- Aspiration
c- Rupture of suture line
d- Met. Alkalosis
152- The nurse is discussing dietary sources of iron with a client who has
iron deficiency anemia. Which menu, if selected by the client, indicates the
best understanding of the diet?
a- Milkshake, hot dog, and beets
b- Beef steak, spinach, and grape juice
c- Chicken salad, green peas, and coffee
d- Macaroni and cheese, coleslaw, and lemonade
153- A client is recovering from abdominal surgery and has a large
abdominal wound. A nurse encourages the client to eat which food item
that is naturally high in vitamin C to promote wound healing:
46
a- Milk
b- Oranges
c- Bananas
d- Chickens
154- Which of the following reflexes is considered a normal reflex of the
newborn:
a- Moro reflex
b- Rooting reflex
c- Planter reflex
d- Tonic neck reflex
155- Patient with aortic aneurysm, which of the following is the most
dangerous complications:
a- Embolism
b- Rupture
c- Stenosis
d- Hypotension
156- 56 years old patient suffers from heart failure, when the nurse
examines his lungs she noticed the presence of crackles which means that
the patient has:
a- Right-sided- failure
b- Left-sided- failure
c- Biventricular failure
d- Congestive heart failure
157- The nurse teaches a client with chronic obstructive pulmonary disease
(COPD) to assess for signs and symptoms of right-sided heart failure.
47
Which of the following signs and symptoms should be included in the
teaching plan:
a- Clubbing of nail beds
b- Hypertension
c- Peripheral edema
d- Increased appetite
158-A surgery procedure that must be done within 24 hours is called:
a- Selective surgery
b- Elective surgery
c- Urgent surgery
d- Emergency surgery
159-Early sign of ARDS in a patient at risk:
a- Elevated CO2 level
b- Hypoxia not responsive to O2 (cyanosis)
c- Metabolic acidosis
d- Severe, unexplained light imbalance
160- A nurse is assessing a female client with multiple trauma who is at risk
for developing acute respiratory distress syndrome. The nurse assesses for
which earliest sign of acute respiratory distress syndrome:
a- Bilateral wheezing
b- Inspiratory crackles
c- Intercostal retractions
d- Increased respiratory rate
161-36 years old male patient complaining of fever and headache for 3 days
now, when examining this patent which of the following signs would
indicate that this patient may has meningitis:
48
a- Positive Kernig's sign
b- Negative Brodzinski's sign
c- Positive homan's sign
d- Negative Kernig's sign
162-A patient presented with high fever, headache, vomiting and neck
stiffness for the past 3 days, which of the following is the first diagnostic
intervention for this patient:
a- Urine and stool analysis
b- Lumber puncture with CSF aspiration
c- Complete blood count
d- Chest and abdomen x-ray
163-MRSA (methicillin resistant Staphylococcus aureus) is transmitted by:
a- Contact
b- Airborne
c- Droplet
d- Standard
164- What is the best indicator of effective treatment of CHF in patient
taking Lasix?
a- Calculating total intake daily
b- Weighting himself daily
c- Calculating urine output daily
d- Assessing fluid status daily
165- What supplemental medication is most frequently ordered in
conjunction with furosemide (Lasix)?
a- Chloride
49
b- Digoxin
c- Potassium
d- Sodium
166- What is the first intervention for a client experiencing myocardial
infarction?
a- Administer morphine
b- Administer oxygen
c- Administer sublingual nitroglycerine
d- Obtain an electrocardiogram
167- Which type of medications is commonly used to treat rheumatoid
arthritis?
a- Glucocorticoids
b- Non- steroidal ant- inflammatory drugs (NSAIDs)
c- Antimalarial drugs
d- Gold salt
168-A male client undergoes total gastrectomy. Several hours after surgery,
the nurse notes that the client's nasogastric (NG) tube has stopped draining.
How should the nurse respond?
a- Notify the physician
b- Reposition the tube
c- Irrigate the tube
d- Increase the suction level
169-Postoperatively, patient is expected to void after:
a- 6-8 hours
50
b- 2-4 hours
c- 12-24 hours
d- 10-12 hours
170- A female client has severe menstrual pain, which of the following
drugs you would recommend for this patient to relieve her dysmenorrhea:
a- Ibuprofen
b- Zantac
c- Aspirin
d- Cortisone
171- The nurse must suction a child with a tracheostomy. Interventions
should include:
a- Encouraging the child to cough to raise the secretions before suctioning
b- Selecting a catheter with a diameter three fourths as large as the diameter
of the tracheostomy tube
c- Ensuring that each pass of the suction catheter take no longer than 5
seconds
d- Allowing the child to rest after every five times the suction catheter is
passed
172-A 40 years old female complains of abdominal pain. It is worse after
eating, especially if she has a meal that is spicy or high in fat. She has tried
over-thecounter antacids, but they have not helped the pain. After
examining her abdomen, you strongly suspect cholecystitis. Which sign on
examination increases your suspicion for this diagnosis:
a- Psoas sign
b- Rovsing' s sign
51
c- Murphy's sign
d- Grey turner's sign
173- Which pulse should the nurse palpate during rapid assessment of an
unconscious male adult?
a- Radial
b- Brachial
c- Femoral
d- Carotid
174- The most important nursing measure in the prevention of
thrombophlebitis for the post-partum mother is:
a- Elastic stocking
b- Early ambulation
c- Anticoagulants
d- Isometric exercises
175- Which of the following is the best aid to prevent breast cancer:
a- Teaching women about breast cancer
b- Public knowledge about chemotherapy
c- To eat fruits and vegetables only
d- Encourage women to perform self-breast examination monthly
176- Which of the following factors would contribute to a high risk
pregnancy:
a- Blood type 0 positive
b- First pregnancy at 33 years old
c- History of allergy to honey bee pollen
d- History of insulin dependent DM
52
177- When the nurse puts an infant with high temperature in cold water in
order to lower his temperature, she knows that the mechanism of heat loss
is called:
a- Evaporation
b- Conduction
c- Radiation
d- Convection
178- A child is admitted to the hospital with a diagnosis of Wilm's tumor,
Stage D. Which of the following statements most accurately describes this
stage:
a- The tumor is less than 3 em. in size and requires no chemotherapy
b- The tumor did not extend beyond the kidney and was completely resected
c- The tumor extended beyond the kidney but was completely resected
d- The tumor has spread into the abdominal cavity and cannot be resected
179- When assessing a 12 year old child with Wilm's tumor, the nurse
should keep in mind that it most important to avoid which of the following:
a- Measuring the child's chest circumference
b- Palpating the child's abdomen
c- Placing the child in an upright position
d- Measuring the child's occipitofrontal circumference
53
180- The nurse is teaching a 45 year old woman how to increase the
potassium in her diet. The woman says she knows bananas are high in
potassium, but she doesn't like their taste. What foods should the nurse
recommend the client include in her diet:
a- Carrots, broccoli, yogurt
b- Rhubarb, tofu, celery
c- Potatoes, spinach, raisins
d- Onions, com, oatmeal
181- When developing a plan of care for a patient with SIADH (syndrome
of inappropriate antidiuretic hormone secretion), which interventions will
the nurse include:
a- Encourage fluids to 2000 ml/day
b- Long-term fluid restriction
c- Monitor for increased peripheral edema
d- Keep head of bed elevated to 30 degrees
182-A client is admitted for treatment of the syndrome of inappropriate
antidiuretic hormone (SIADH). Which nursing intervention is appropriate?
a- Infusing LV. fluids rapidly as ordered
b- Administering glucose-containing LV. fluids as ordered
c- Restricting fluids
d- Encouraging increased oral intake
183-How streptococci infection is transmitted from person to another:
a- Airborne
b- Contact
c- Slandered
54
d- Blood
184-A hospitalized client is found to be comatose and hypoglycemic with a
blood sugar of 50 mg/dL. Which of the following would the nurse do first?
a- Infuse 1000 mL D5W over a 12-hour period
b- Encourage the client to drink orange juice with added sugar
c- Check the client's urine for the presence of sugar and acetone
d- Administer 50% glucose intravenously
185- A patient receives TPN, which of the following nursing actions is
important to minimize the risk for fluid volume excess:
a- Increase diuretic dose if swelling occurs
b- Limit the amount of free water in relation to sodium intake
c- Monitor his or her skin turgor
d- Weigh the patient daily on the same scale
186- Which of the following findings would most likely indicate the
presence of a respiratory infection in a client with asthma:
a- Cough productive of yellow sputum
b- Bilateral expiratory wheezing
c- Chest tightness
d- Respiratory rate of 30 breaths/ minute
187- When teaching a mother how to administer eye drops, where should
the nurse tell her to place them?
a- In the conjunctival sac that is formed when the lower lid is pulled down
b- Carefully under the eye lid while it is gently pulled upward
c- On the sclera while the child looks to the side
d- Anywhere as long as drops contact the eye's surface
55
188- One nursing intervention for patient with asthma is to facilitate
removal of secretions. This can be done by:
a- Encourage the patient to perform slow and shallow breathing
b- Encourage the patient to increase fluid intake
c- Encourage the patient to hyperventilate
d- Encourage the patient to decrease fluid intake
189- The nurse is caring for a 6-year-old boy following revision of
ventricuioperitoneal shunt. An expected nursing intervention is:
a- Request for an x-ray to evaluate shunt placement
b- Daily measurement of head circumference
c- Frequent palpation of the fontanels
d- Maintaining the child in a prone position
190-the patient have an itchy, erythematous, vesicular, weeping, and
crusting patches on his skin which skin disease is this considered:
a- Psoriasis
b- Impetigo
c- Eczema
d- Urticaria
191-Giving an intramuscular injection to an infant in the Gluteus Maximus
muscle may produce an injury to which of the following nerves:
a- Femoral nerve
b- Sciatic nerve
c- Vagus nerve
d- Popliteal nerve
56
192- Which of the following is one of the side effects of albuterol nebulizer:
a- Hypertension
b- Hypotension
c- Fast irregular heart beats
d- Tachypnea
193- A patient, age 49, returns from the postanesthesia care unit after a
total abdominal hysterectomy and bilateral salpingo-oophorectomy to treat
cervical cancer. Which nursing intervention has the highest priority at this
time?
a- Monitor the patient for indications of hemorrhage
b- Assess the patient's pain level and response to analgesics
c- Encourage the patient to do deep breathing and leg exercises
d- Provide emotional support to the patient
194- When planning the care of a client who has undergone an abdominal
hysterectomy, which nursing measure is most helpful for preventing
postoperative complications and facilitating an early discharge?
a- Reestablishing oral fluids and nutrition
b- Promoting ambulation and movement
c- Maintaining accurate intake and output
d- Exploring feelings about altered image
195- Which of the following is considered one of the common complications
following abdominal total hysterectomy:
a- DVT
b- Paralytic ileus
c- Constipation
d- Perforation
57
196-A client had a hysterectomy 10 hours ago. The nurse assesses the client
and finds that her blood pressure has fallen abruptly. Which action by the
nurse is most appropriate at this time?
a- Continue to monitor the blood pressure every 15 minutes
b- Document the information on the client's chart
c- Inform the surgeon about the client's condition
d- Change the client to a Fowler's position
197- The nurse is teaching a client regarding risk factors for stroke (CVA).
The greatest risk factor is which of the following:
a- Diabetes
b- Heart disease
c- Renal insufficiency
d- Hypertension
198- The nurse recognizes that the most common type of brain attack
(CVA) is related to which of the following:
a- Ischemia
b- Hemorrhage
c- Headache
d- Vomiting
199-A woman comes to the clinic and states, "I've been sick for so long! My
eyes have gotten so puffy, and my eyebrows and hair have become coarse
and dry." The nurse will assess for other signs and symptoms of:
a- Cachexia
b- Parkinson's syndrome
c- Myxedema
d- Scleroderma
58
200- A 32 year-old man is brought to the emergency department by a friend
with whom he had been playing football with earlier that evening. His
symptoms came on after twenty minutes of the football match. He appears
anxious and restless. Auscultation of the lung reveals wheezing on
exhalation. The patient states that he is unable to get a full breath of air. He
had this problem as a child a couple of times after exercising in cold
weather.
Blood pressure
126/72 mmhg
Heart rate
90/min
Respiratory rate
28/min shallow
Temperature
37.0C tympanic
Oxygen saturation
94% on room air
Which position is the best for this patient?
a- Supine
b- Reverse Trendelenburg's
c- Prone
d- High fowler's
201- A 43 years- old man is 30- hour post- operative following placement of
a partial thickness skin autograft for a burn injury on the lower anterior
leg. During a routine assessment, the nurse observes the wound is bleeding
continuously.
What is the priority nursing action?
a- Use a pen to outline and monitor the area
b- Perform a wound swab for laboratory analysis
c- Incise and drain fluids from the wound bed
d- Apply firm and direct pressure for 10 minutes
59
202-Coarse hair may indicate:
a- Dehydration
b- Inflammation
c- Under nutrition
d- Anemia
203- The nurse performs a home visit for a 32 years- old woman who had
given birth to her first infant three days before. The mother has concerns
about breastfeeding and the nurse observes the infant feeding. The mother
sits supported upright with cushions and the infant positioned in a cradle
hold. The infant's head and body are aligned against the mother's
abdomen. The infant sucks intermittently with the lips turned outwards
and the nipple in the mouth.
Which intervention is most appropriate?
a- Reposition the infant
b- Assist the infant to turn lips inwards
c- Ensure the latch includes the areola
d- Stimulate the infant to suck constantly
204- A 52 year- old woman is scheduled to undergo an abdomino- perineal
resection in three days for removal of a cancer of the rectum. The nurse
reviews the care plan with the patient. The patient will receive prophylactic
antibiotics and will be given a mechanical bowel preparation the day
before.
Which additional preparation should the patient undertake at this time?
a- Wear pressure stockings
b- Perform leg strengthening exercises
c- Maintain high- protein, low- residue diet
60
d- Take daily ferrous iron tablets
205-A patient is preparing for a scheduled hip replacement.
Which lab value should be reported to the physician?
Test
Sodium
Potassium
Calcium
Magnesium
Result
145
2.9
2.80
4.8
Normal values
134-164 mmol/L
3.5-5.1 mmol/L
2. 15-2.62mmol/L
1.2-2 mmol/L
a- Sodium
b- Calcium
c- Potassium
d- Magnesium
206- A nurse is admitting a six month- old infant with pneumonia. Which of
the following interventions supports this infant's emotional needs?
a- Allow the parents to leave the room during painful procedures
b- Encourage parents to distract the infant from crying
c- Interview the patents to learn the infant's comforting habits
d- Enforce strict visiting schedule and routines
207- While caring for a patient in the post- anesthesia care unit (PACU)
who has developed hypovolemic shock, a nurse should position the patient:
a- Flat with legs elevated
b- In trendelenburg position
c- With the head of the bed elevated 45 degrees
61
d- Completely flat
208- A five years - old child girl had presented with tenderness, headache
and petechiae. She was pale and complains of joint pain. On palpation there
was an enlarged spleen, liver and lymph nodes. A lumbar puncture showed
central nervous system involvement. The child underwent chemotherapy
treatment and is now attended by the nurse for regular routine
examinations.
Which condition would require more attention from the nurse?
a- Increased leukocytes
b- Lack of muscle coordination
c- Bleeding while brushing teeth
d- Occasional nausea and vomiting
209-During the evaluation at a community clinic, the patient completes the
medical history. Which of the following is NOT a risk factor for an acute
myocardial infarction?
a- Coronary artery disease
b- Smoking
c- Hemophilia
d- Hyperlipidemia
210- A postoperative patient has the nursing diagnosis of ineffective tissue
perfusion. To assess for tissue perfusion the nurse should check all of the
following except:
a- Skin and nail bed color
b- Temperature of extremities
c- Respiratory rate
62
d- Peripheral pulses
211-A patient visits the clinic for a 2- week checkup after a corneal
transplantation (keratoplasty). The nurse observes the patent's sclera is red
and the patient complains of the eye feeling irritated. The nurse suspects
the patient may have:
a- Infection
b- Hemorrhage
c- Graft rejection
d- Postoperative glaucoma
212- The nurse is assessing a patient who just had surgery under general
anesthesia. The patient's respiration rate is 4 per minute and the O2
saturation on 3L per minute of oxygen via nasal cannula is 84%. The nurse
is awaiting the results of an arterial blood gas (ABG) and anticipates that
which of the following will be ELEVATED?
a- Arterial oxygen saturation (SaO2)
b- Hydrogen ion concentration (PH)
c- Partial pressure of arterial oxygen (PaO2)
d- Partial pressure of arterial carbon dioxide (PaCO2)
213-During the immediate postoperative period, a patient reveals an oxygen
saturation of 91 %. The nurse should:
a- Position the patient on the left side
b- Administer supplemental oxygen
c- Continue to provide supportive care
63
d- Lower the temperature of the room
214- The nurse performs an assessment of a 23 year- old man who believes
that people are spying on him. During the interview, he keeps his eyes to the
floor and answers questions awkwardly. He has never had an intimate
relationship and avoids contact with his family members. He has never been
employed and tells the nurse that he is not looking for a job. The nurse
considers Erickson's theory of psychosocial development.
Which stage is this patient most likely experiencing?
a- Autonomy versus shame and doubt
b- Initiative versus guilt
c- Trust versus mistrust
d- Identity versus confusion
215-A 32 year- old woman with a motor neuron disease has progressive
weakness of cranial nerves V, IX, XII. She has recently been experiencing
sudden outbursts of crying and laughing. Both stroke and myasthenia
gravis have been ruled out.
Which voluntary muscle activity would be most significantly impacted?
a- Walking
b- Swallowing
c- Breathing
d- Smelling
216- Causes of primary hypothyroidism in adults include:
a- Malignant or benign thyroid nodules
b- Surgical removal or failure of the pituitary gland
c- Surgical removal or radiation of thyroid gland
d- Autoimmune-induced atrophy of the gland
64
217-A 72-year-old patient is diagnosed with hypothyroidism, and
levothyroxine (Synthroid) is prescribed. During initiation of thyroid
replacement for the patient, it is most important for the nurse to assess:
a- Mental status
b- Nutritional level
c- Cardiac function
d- Fluid balance
218-A client presents with Hypocalcemia, hyperphosphatemia, muscle
cramps, and positive Trousseau's sign. What diagnosis does this support?
a- Diabetes insipidus
b- Conn's syndrome
c- Hypoparathvroidism
d- Acromegaly
219- The nurse acknowledges that the first-line drug for treating of client's
blood pressure might be which drug:
a- Diuretic
b- Alpha blocker
c- ACE inhibitor
d- Alpha/beta blocker
220- The nurse is teaching a client about clopidogrel (plavix). What is
important information to include?
a- Constipation may occur
b- Hypotension may occur
c- Bleeding may increase when taken with aspirin
d- Normal dose is 25 mg tablet per day
65
221- The nurse knows that the client's cholesterol level should be within
which range:
a- 150 to 200 mg/dL
b- 200 to 225 mg/dL
c- 225 to 250 mg/dL
d- Greater than 250 mg/dL
222- The nurse is performing an assessment in a client with a suspected
diagnosis of cataract. The chief clinical manifestation that the nurse would
expect to note in the early stages of cataract formation is:
a- Eye pain
b- Floating spots
c- Blurred vision
d- Diplopia
223- A client who has a history of Crohn's disease is admitted to the
hospital with fever, diarrhea, cramping, abdominal pain, and weight loss.
The nurse should monitor the client for:
a- Hyperalbuminemia
b- Thrombocytopenia
c- Hypokalemia
d- Hypercalcemia
66
224-Adult patient complains of diarrhea, vomiting, abdomen cramp and
pain within the past 2 weeks. The patient reported that the pain increases
when he eats and relieves when he passes stool. Which of the following may
be the cause:
a- Appendicitis
b- Crohn' s disease
c- Ectopic pregnant
d- Cholecystitis
225- When caring for a 3-day-old neonate who is receiving phototherapy to
treat jaundice, the nurse in charge would expect to do which of the
following:
a- Turn the neonate every 6 hours
b- Encourage the mother to discontinue breast-feeding
c- Notify the physician if the skin becomes bronze in color
d- Check the vital signs every 2 to 4 hours
226- After 3 days of breast-feeding, a postpartum patient reports nipple
soreness. To relieve her discomfort, the nurse should suggest that she:
a- Apply warm compresses to her nipples just before feedings
b- Lubricate her nipples with expressed milk before feeding
c- Dry her nipples with a soft towel after feedings
d- Apply soap directly to her nipples, and then rinse
227- Which of the following interventions would be helpful to a
breastfeeding mother who is experiencing engorged breasts:
a- Applying ice
b- Applying a breast binder
c- Teaching how to express her breasts in a warm shower
d- Administering bromocriptine (Parlodel)
67
228- What type of milk is present in the breasts 7 to 10 days postpartum?
a- Colostrum
b- Hind milk
c- Mature milk
d- Transitional milk
229- Which of the following is a sign of rejection after a renal
transplantation operation:
a- Decrease potassium
b- Decrease sodium
c- Decrease creatinine
d- Decrease urine out put
230- Which of the following is among the signs of magnesium toxicity:
a- Cardiac arrhythmia
b- Loss of deep tendon reflex
c- Patient confusion and irritability
d- Respiratory failure
231-A nurse is caring for a client who has a potassium level of 5.4 mEqlL.
The nurse should assess the client for:
a- ECG changes
b- Constipation
c- Polyuria
d- Hypotension
68
232-Moderate to severe hypokalemia in a patient will cause the nurse to
observe:
a- Muscle spasms and slow respirations
b- Muscle weakness and cardiac dysrhythmias
c- Confusion and irritability
d- Vomiting and diarrhea
233- Which of the following electrolytes decrease would cause PVC
(premature ventricular contraction):
a- Potassium
b- Sodium
c- Calcium
d- Magnesium
234- Which of the following electrolytes decrease would cause recurrent
vomiting and general weakness:
a- Potassium
b- Sodium
c- Calcium
d- Magnesium
235- The nurse is educating a couple who has had difficulty with
conception. The client asks about the hormone, which is responsible for the
production of eggs. The nurse answers that this hormone is called:
a- Melanocyte-stimulating hormone
b- Luteinizing hormone (LH)
c- Follicle-stimulating hormone (FSH)
d- Interstitial cell-stimulating hormone (ICSH)
69
236- The patient has an order to receive 45 mg of prednisone by mouth
daily. Available are 10-mg tablets. How many tablets should the nurse
prepare to give?
a- 1.6
b- 2
c- 4.5
d- 6
237- The nurse caring for a 54-year-old patient hospitalized with diabetes
mellitus would look for which of the following laboratory test results to
obtain information on the patient's past glucose control:
a- Prealbumin level
b- Urine ketone level
c- Fasting glucose level
d- Glycosylated hemoglobin level
238- Which laboratory test is most important for the nurse to monitor to
determine how effectively the client's diabetes is being managed?
a- Fasting blood glucose
b- Blood chemistry profile
c- Complete blood count
d- Glycosylated hemoglobin (HbAlc)
239-Blood sugar for new employee at company with sedentary life style and
obese and by result of Fasting blood Sugar 6.9 mmol (abnormal reading)
what is the best next step to do to estimate his blood sugar condition:
a- Draw blood sample for HbAlc
b- Repeat the test for the next 2 days
c- Estimate the postprandial sugar level
d- Write a recommendation note for not hiring him
70
240- HbA1c as a measurement instrument for glucose level in the blood, it
measures blood glucose level within:
a- Last 3 months
b- Last 5 months
c- Last 7 months
d- Last 1 year
241- Which laboratory test indicates liver cirrhosis?
a- Decreased red blood cell count
b- Decreased serum acid phosphatase level
c- Elevated white blood cell count
d- Elevated serum aminotransferase
242- Which statement best describes the difference between the pain of
angina and the pain of myocardial infarction:
a- Pain associated with angina is relieved by rest
b- Pain associated with myocardial infarction is always more severe
c- Pain associated with angina is confined to the chest area
d- Pain associated with myocardial infarction is referred to the left arm
243- The nurse is developing a bowel-retraining plan for a client with
multiple sclerosis. Which measure is likely to be least helpful to the client?
a- Limiting fluid intake to 1000mL per day
b- Providing a high-roughage diet
c- Elevating the toilet seat for easy access
d- Establishing a regular schedule for toileting
71
244- Which organism is responsible for the onset of rheumatic fever?
a- Staphylococcus aureus
b- Listeriosis
c- Group A β-hemolytic streptococcus
d- Epstein-Barr virus
245-For what reason are patients instructed to do Kegel exercises when
they are discharged after childbirth:
a- They help her get back in shape
b- They prevent her breasts from becoming engorged
c- They help strengthen the perineal muscles
d- They help reduce strain on the back
246- You are preparing to discharge a patient who suffered third degree
burn, which of the following instructions is appropriate to give the patient
to decrease infection:
a- You must take large amount of fluids
b- You should wash your hands thoroughly
c- You better increase vitamin intake
d- You should avoid fatty meals
247- The most effective procedure to prevent spared of infection is:
a- Wearing gloves
b- Using antiseptic
c- Sterilization
d- Hand washing
72
248- What is the normal pulse range for an adult?
a- 120 to 160 beats per minute
b- 90 to 140 beats per minute
c- 60 to 100 beats per minute
d- 50 to 80 beats per minute
249- A four year- old girl was playing outside in a park when she came
running to her mother crying and holding her right, upper arm. Two hours
later they presented to the emergency department. There was a swelling
over the upper right arm, with pain and itching at the site as well as
swelling of the oral mucosa. The child seems anxious.
What is the next most appropriate step in management?
a- Administer subcutaneous epinephrine
b- Maintain a patent airway
c- Administer oral diphenhydramine
d- Prepare intubation equipment
250- A 65 year- old man presents with a resting tremor in the right
forearm. An assessment of gait reveals decreased arm swinging and slight
dragging of the foot on the right side. His body movements are slow. He has
not been sleeping well at night for the past six months due to leg pain and
says that he feels constantly tired and weak. He reports that he has not
suffered any recent fall and that the symptoms seem to be slowly worsening.
Which medication is most likely to be administered?
a- Levodopa
b- Haloperidol
c- Phenytoin
d- Benzodiazepine
73
251- When planning discharge teaching for the parent of an infant with
bronchiolitis, the nurse should EMPHASIZE:
a- Use of supplemental oxygen at night
b- Frequent hand washing
c- Sleeping in the supine position
d- Rice- thickened formula during night- time feedings
252-A home care nurse visits a diabetic patient who was started on insulin
injections. Upon examination, the nurse observes small lumps and dents on
the right upper arm where the patient has injected insulin.
What is the BEST nursing intervention?
a- Refer the patient to a dermatologist for diabetic cellulitis
b- Instruct the patient to rotate the sites of injection
c- Refer the patient to an endocrinologist for better control of glucose level
d- Instruct the patient to inject in the muscular area instead of the
subcutaneous area
253-A patient has an order for 1000 milliliters (ml) of intravenous (IV) fluid
to infuse over eight hours. The available IV tubing has a drip factor of 10
gtts/ml.
Which of the following rates is correct?
a- 125 ml/hour
b- 125 drop/minute
c- 21 drops/minute
d- 21 ml/hour
74
254- When is the correct time for the nurse to administer the child's
morning dose of a combination regular and NPH insulin?
a- 30 minutes before breakfast is served
b- 15 minutes before breakfast is served
c- 30 minutes after breakfast is served
d- 15 minutes after breakfast is served
255- The client has an order for administration of 10 units of regular
insulin to be given at 7:00 a.m. The nurse should offer a snack at:
a- 3:00 p.m.
b- 1:00 p.m.
c- 11:00 a.m.
d- 9:00 a.m.
256- The client is being treated with PH insulin at 8:00 a.m. The nurse
should offer a snack at:
a- 9:00 a.m.
b- 11:00 a.m.
c- 6:00 p.m.
d- 2:00 p.m.
75
257-A client with diabetes comes to the emergency department. The nurse
obtains a blood glucose measurement with a glucometer and notes that it is
510 mg/dL. The physician orders I.V. insulin. Which type of insulin can be
given both intravenously and subcutaneously?
a- Regular
b- NPH
c- Lente
d- 70/30
258-1f a client with type 1 (insulin-dependent) diabetes mellitus receives 5
units of NPH insulin every morning at 7 a.m., the nurse should closely
monitor the client for signs of hypoglycemia at what time:
a- 3:00 p.m.
b- 12:00 a.m.
c- 7:30 a.m.
d- 10:00 p.m.
259- NPH is an example of which type of insulin:
a- Short-acting
b- Intermediate-acting
c- Rapid-acting
d- Long-acting
76
260- A 70 years- old woman presents with increasing dyspnea on exertion.
She feels breathless and restless while performing household tasks, such as
making the bed and sweeping the floor. Her previous medical history
includes a myocardial infarction at 57 years old. She sleeps with her head
elevated on three pillows. Examination reveals bilateral basal crackles and
cold, damp skin.
Blood pressure
172/94 mmhg
Heart rate
94/min
Respiratory rate
36/min
Temperature
37.1 C
Oxygen saturation
90% on room air
Which heart chamber most likely failed first?
a- Right atrial
b- Right ventricle
c- Left ventricle
d- Left atrial
261- The nurse is preparing a discharge plan for a 65 year- old man with a
new diagnosis of congestive heart failure. The discharge orders include
furosemide 40 milligrams by mouth twice per day. The nurse recommends
food to reduce unwanted medication side effects.
Which of the following food would be most appropriate to include in the
teaching plan?
a- Green and leafy vegetables
b- Bananas and oranges
c- Chicken and fish
d- Whole wheat grains
77
262- A child comes into the clinic with several lesions to the scalp. The
round lesions have dandruff like scaling with hair loss.
What is the MOST likely diagnosis for this skin condition?
a- Impetigo
b- Ringworm (tinea capitis)
c- Pediculosis capitis
d- Scabies
263- A 40 years- old woman suffered an automobile accident one month
ago. The accident resulted in a C-4 spinal cord injury. Her preventive care
includes independent daily performance of coughing and deep breathing.
Which range- of- motion exercise would be most beneficial for this patient?
a- Active
b- Passive
c- Combined
d- Resistive
264- A nine year- old girl has a nursing diagnosis of altered body image
related to changes in appearance secondary to varicella infection. The
child's body is covered with a rash and many large, weeping pustules. The
nurse provides counseling to the mother who is concerned that the child will
develop scarring.
Which intervention is most appropriate?
a- Keep the skin out of direct sunlight
b- Apply calamine lotion to skin every two hours
c- Wash pustules with soap and keep dry
d- Soak in colloidal oatmeal bath three times daily
78
265-A nurse is assessing a client with possible Cushing's syndrome. In a
client with Cushing's syndrome, the nurse expects to find:
a- Weight gain in arms and legs
b- Thick, coarse skin
c- Hypotension
d- Deposits of adipose tissue in the trunk and dorsocervical area
266- Which vitamin deficiency is most likely to be a long-term consequence
of a full-thickness burn injury?
a- Vitamin A
b- Vitamin B
c- Vitamin C
d- Vitamin D
267-Sildenafil (Viagra) is prescribed to treat a client with erectile
dysfunction. A nurse reviews the client's medical record and would
question the prescription if which of the following is noted in the client's
history:
a- Neuralgia
b- Insomnia
c- Use of nitroglycerin
d- Use of multivitamins
268- The physician ordered I liter of normal saline infusion for four hours,
how much N/S should be delivered to the patient in one hour:
a- 300 ml
b- 250 ml
c- 200 ml
d- 150 ml
79
269- Best diagnostic test for suspected leukemia is:
a- Bone marrow aspiration
b- Blood chemistry
c- CBC
d- PT ANDPTT
270- The primary nursing goal when caring of a child with leukemia is to:
a- Take vital signs
b- Provide good nutrition
c- Give analgesics
d- Prevent infection
271- The oxygen flow rate for the nasal cannula is considered to be:
a- 1-6 liter/m
b- 3-8 liter/m
c- 5 -1 0 liter/m
d- 10-15 liter/m
272 -1f the client develops a thrombus in one of the leg veins, which client
response would the nurse expect when eliciting Homans' sign?
a- Sham, immediate calf pain
b- Sudden numbness in the foot
c- Inability to bend the knee when asked
d- Tingling throughout the affected leg
80
273- When the nurse assesses the client for Homans' sign, which technique
is most accurate?
a- Have the client push each foot against the mattress.
b- Have the client extend the legs and flex each foot toward the head
c- Ask the client to sit up in bed and point all the toes forward
d- Ask the client to contract the thigh muscles
274- The phase that include action of the nursing care plan is called:
a- Implementation
b- Diagnosis
c- Assessment
d- Evaluation
275-After vein ligation and stripping operation, the patient should:
a- Wear elastic stocking
b- Do exercises
c- Rest the leg
d- Avoid walking
276-Rabies disease is considered:
a- Viral disease
b- Bacterial disease
c- Protozoan disease
d- Fungal disease
277-A client receiving IV infusion. The skin around the IV insertion site is
red, warm to touch and painful. The nurse should first:
a- Discontinue the IV
b- Slow the IV rate for 30 minutes then reassess the site
c- Place a cold compression on the area
81
d- Place a warm compression on the area
278-Clients who have casts applied to an extremity must be monitored for
complications. The most significant complication for which the nurse
should assess the client's extremity is:
a- Warmth
b- Numbness
c- Skin desquamation
d- Generalized discomfort
279- A nurse is assessing the legs of a client who's 36 weeks pregnant.
Which finding should the nurse expect?
a- Absent pedal pulses
b- Bilateral dependent edema
c- Sluggish capillary refill
d- Unilateral calf enlargement
280- Which of the following is the best nursing action to assess pulse in a
toddler patient:
a- Assess the brachial artery with the middle finger and index
b- Assess the Dorsalis pedis artery with your palm
c- Assess the carotid artery with the four fingers
d- Assess the radial artery using three finger
281- One nursing intervention for patient with productive cough is to
facilitate removal of secretions. This can be done by:
a- Encourage the patient to perform slow and shallow breathing
b- Encourage the patient to increase fluid intake
c- Encourage the patient to hyperventilate
d- Encourage the patient to decrease fluid intake
82
282-A nurse examines a client's level of responsiveness. She finds that the
patient opens his eyes to verbal commands, obeys verbal commands, and is
oriented to time, place, and person. What's the client's Glasgow Coma
Scale:
a- 11/15
b- 12/15
c- 13/15
d- 14/15
283-A 73 year old patient looks at you when you speak to her. When you
ask her the date, she says "blue". You note left-sided weakness when she
grips your fingers. What is her Glasgow Coma Score?
a- 11/15
b- 12/15
c- 13/15
d- 14/15
284-A patient during examination opens his eyes in response to pain, makes
no verbal response, but withdraws from pain. What is the Glasgow Coma
Score (GCS) for this patient?
a- 3
b- 5
c- 7
d- 11
285-A newborn APGAR score at 1 and 5 minutes is 5 and 10, half an hour
later the baby became bluish in color with heart rate of 140/m, your first
action would be:
a- Estimate the score again
b- Shower the baby with warm water
c- Give oxygen immediately
83
d- Ignore the finding because it is normal
286- The primary critical observation for Apgar scoring is the:
a- Heart rate
b- Respiratory rate
c- Presence of meconium
d- Evaluation of the Mom reflex
287- When performing a newborn assessment, the nurse should measure
the vital signs in the following sequence:
a- Pulse, respirations, temperature
b- Temperature, pulse, respirations
c- Respirations, temperature, pulse
d- Respirations, pulse, temperature
288- Within 3 minutes after birth the normal heart rate of the infant may
range between:
a- 100 and 180
b- 130 and 170
c- 120 and 160
d- 100 and 130
289- The expected respiratory rate of a neonate within 3 minutes of birth
may be as /high as:
a- 50
b- 60
c- 80
d- 100
84
290- When performing nursing care for a neonate after a birth, which
intervention has the highest nursing priority:
a- Obtain a dextrostix
b- Give the initial bath
c- Give the vitamin K injection
d- Cover the neonates head with a cap
291- Which of the following instructions are appropriate for the nurse to
give the patient about the time of taking omeprazole tablet:
a- Take the tablet during the meal
b- Take the tablet 2 hours after the meal
c- Take the tablet 30-45 minute before the meal
d- Take the tablet before bed time
292- Patient obtained second degree burn to his abdomen and his entire
back, according to rule of nine what is the percentage of burn of total body
surface area:
a- 9%
b- 18%
c- 27%
d- 36%
293-R.T.A. Patient arrived to E.R, on examination the patient has increased
heart rate, low blood pressure and decreased level of consciousness, you
must think that the patient is developing:
a- Coma
b- UTI
c- Shock
d- Respiratory arrest
85
294- Which of the following disease has no vaccinations:
a- Small pox
b- Measles
c- Poliomyelitis
d- Chicken pox
295- The type of burn in which all the dermis and epidermis, is destroyed
and there is involvement of underlying structures is called:
a- Superficial or first degree bum
b- Partial thickness or second degree bum
c- Full-thickness or third degree bum
d- Fourth degree bum
296- Which of the following is the drug of choice for treatment of rheumatic
fever:
a- Amoxicillin
b- Azithromycin
c- Cephalexin
d- Penicillin
297- What disease is associated with a Vitamin C deficiency?
a- Pellegria
b- Neural tube defects
c- Scurvy
d- Pitted edema
86
298- A 59 year old woman is diagnosed with Hemolytic anemia. What foods
would be recommended for her to eat:
a- Fish oils
b- Eggs, cheese
c- Yellow fruits
d- Grains, nuts
299- The physician orders an intradermal injection of 5 tuberculin units /0.l
ml of tuberculin purified derivative. Which needle is appropriate for this
injection:
a- 12 gauge needle
b- 18 gauge needle
c- 22 gauge needle
d- 26 gauge needle
300- Which nursing diagnosis takes priority for a newly diagnosed patient
with a left- sided stroke?
a- Risk of impaired swallowing related to absent gag reflex
b- Risk for impaired skin integrity related to immobility
c- Risk for infection related to invasive line placement
d- Risk for impaired speech related to left sided stroke
301- When caring for a patient with left-sided homonymous hemianopsia
resulting from a stroke, which intervention should the nurse include in the
plan of care during the acute period of the stroke?
a- Apply an eye patch to the left eye
b- Approach the patient from the left side
c- Place objects needed for activities of daily living on the patient's right side
d- Reassure the patient that the visual deficit will resolve as the stroke
progresses
87
302- A client with a serum glucose level of 618mg/dl is admitted to the
facility. He's awake and oriented, has hot dry skin, and has the following
vital signs:
Temperature
38.10 C
Heart rate
116 beats/minute
Blood pressure
108/70 mm Hg
Based on these assessment findings, which nursing diagnosis takes the
highest priority?
a- Deficient fluid volume related to osmotic diuresis
b- Decreased cardiac output related to elevated heart rate
c- Imbalanced nutrition: less than body requirements related to insulin
deficiency
d- Ineffective thermoregulation related to dehydration
303- A patient has a blood glucose level of 60mg/dl. The patient is to receive
15 g of carbohydrate if the blood glucose is less than 70mg/dl.
How many mls of orange juice should the patient receive?
a-
120 ml
b-
90 ml
c-
60 ml
d-
30 ml
88
304-A patient has a blood glucose level of 60mg/dl. The patient is to receive
15 g of carbohydrate if the blood glucose is less than 70mg/dl.
How many candies should the patient receive?
a- 4 candies
b- 6 candies
c- 8 candies
d- 10 candies
305- A patient is scheduled for a pneumonectomy in the morning.
Which of the following diagnosis is the MOST likely indication for this type
of surgery?
a- Lung carcinoma
b- Pulmonary tuberculosis
c- Benign pulmonary nodule
d- Mediastinal shift
306- The nurse is caring for a patient with a deep vein thrombosis (DVT).
The patient's heparin sodium infusion has been discontinued and the
patient is receiving prescribed warfarin sodium (Coumadin)
The nurse should advise the patient that which of the following needs to be
continued?
a- Daily complete blood count (CBC)
b- Laboratory tests for partial thromboplastin time (PTT)
c- Strict Bedrest
d- Wearing elasticized support stockings
89
307-A patient receiving a total parenteral nutrition through a central line
suddenly has difficulty breathing and is restless. Chest auscultation reveals
a heart murmur.
Blood pressure
90/60 mmhg
Heart rate
120/min
Respiratory rate
22/min
Temperature
37.1 C
Oxygen saturation
90% on room air
What is the most appropriate initial nursing action?
a- Notify the physician
b- Administer 100% oxygen by face mask
c- Place patient in left side lateral decubitus
d- Obtain stat blood glucose measurement
308-A 16 year- old boy fractured his right tibia and fibula during a football
match. Eight weeks later, the fractures were successfully healed and the
cast was removed.
Which range- of- motion exercises would be most beneficial?
a- Active
b- Passive
c- Combined
d- Resistive
90
309- A 65 year- old woman who was diagnosed with hypothyroidism at the
age of 45 is brought to the clinic by ambulance. On arrival, she had a
severely decreased level of consciousness. Her breathing is shallow and
irregular. The skin is cool, dry and pale. There is generalized non- pitting
edema of all extremities and face.
Blood pressure
70/40 mmhg
Heart rate
60/min
Respiratory rate
121 min
Body temperature
35.5 C tympanic
What is the most appropriate method to re-warm this patient?
a- Warmed intravenous isotonic fluids
b- Place the patient is a warm bath
c- Apply heat packs to the head and neck
d- Cover the patient with hospital blankets
310- The physician ordered a blood glucose test for the neonates the nurse
knows the best site to puncture is usually:
a- The lateral heel
b- Anterior sole
c- Finger tip
d- Anterior scalp
91
311- The nurse is teaching a patient who was recently diagnosed with
rheumatoid arthritis affecting the hands.
Which of the following treatment should the nurse discuss with the patient
at this time?
a- Transcutaneous electric nerve stimulation (TENS)
b- Iontophoresis
c- Nonsteroidal anti-inflammatory drugs (NSAIDs)
d- Synovectomy
Problems
A Altered tissue perfusion related to
hypertension
Fluid volume excess related to lower
B
extremity edema
C Knowledge deficit related to disease
process, hyper tension
D Alternation in comfort related to
chronic headaches
The patient will
Maintain blood pressure below 130/90 mmhg
Not have signs of edema or unexpected weight
gain
Verbalize understanding of disease process,
symptoms and medication management
Report an absence of headaches each week
312-A patient has come to the clinic for follow- up one week after being
discharged from the hospital for treatment of a hypertensive crisis. Blood
pressure stabilized at 124/78 mm Hg. The patient reports feeling well, has
no edema, no longer has daily headaches. Blood pressure is 156/90 mm Hg.
During evaluation the patient admits to having stopped taking medication
that had been ordered because headaches are no longer present. Unless the
symptoms return, the patient states he will not be returning to the clinic.
What should the nurse do?
a- Resolve and discontinue the entire care plan per patient request, suggest
psychology consult
b- Resolve Band D; continue A and C
c- Continue entire care plan as written
d- Add a new problem to the care plan, non-compliance and interventions to
determine potential reasons.
92
313- A 34 year- old quadriplegia patient resides at home with his wife. In
order to prevent contractures of all extremities, the community care nurse
will instruct the patient's wife in performance of:
a- Active range of motion exercises
b- Passive range of motion exercises
c- Active- assistive range of motion exercises
d- Resistive range of motion exercises
314-A six years- old patient has presented to the clinic with fever, malaise,
and anorexia. The patient was treated 2 weeks ago for a streptococcal
infection of the throat.
The nurse should expect the physician to order what test?
a- Electrocardiogram
b- Jones test
c- Spinal tap
d- Heart biopsy
315-A 40 year- old woman presented with right hip pain. Palpation of the
pelvic girdle is normal. An X- ray shows bone deformities, with osteolytic
lesions and bone enlargement. The patient has not suffered any trauma and
has been generally healthy.
Which serum laboratory analysis would be most useful?
a- Prothrombin time
b- Alkaline phosphatase
c- Acid phosphatase
d- Parathyroid hormone
93
316-A client complains of itching about 20 minutes after he began his blood
infusion, which of the following is the best nursing action his caring nurse
must take:
a- Stop the infusion immediately and call the physician
b- Call the physician immediately and the stop the infusion
c- Culminate the client and continue infusion
d- Continue the infusion while monitoring the client closely
317- A 32 year- old woman with diabetes mellitus type 1 underwent a
cholecystectomy and is now on day two of recovery. The patient's bowel
sounds have returned and she has resumed a normal diet but has been
finishing less than half of each meal on the tray. The nurse enters the room
to perform a routine assessment and finds the patient confused and shaky
Blood pressure
110/60 mmhg
Heart rate
96/min
Respiratory rate
22/min
Temperature
37.0 Coral
What is the most appropriate initial intervention?
a- Administer glucagon
b- Notify the physician
c- Give an insulin injection
d- Offer a glass of juice
318-A Cancer patient wants to stop chemotherapy and change to palliative
care, this situation indicates which psychological status:
a- Depression
b- Hopelessness
c- Anxiety
d- Restlessness
94
319- What do we call the type of treatment of terminal illness patients?
a- Conservative
b- Curative
c- Palliative
d- Selective
320-Psychiatric patient appear violent for himself and others was put in the
room alone during the period of exacerbation, then patient calm down and
informed the nurse I am ok now let me with others, but the nurse refused
that as a punishment way. At which underline Label will nurse accused
under the court:
a- Abandonment
b- False imprisonment
c- Negligence
d- Duty to act
321- Which of the following actions is the first priority of care for a client
exhibiting signs and symptoms of coronary artery disease?
a- Decrease anxiety
b- Enhance myocardial oxygenation
c- Administer sublingual nitroglycerin
d- Educate the client about his symptoms
322- What is the first intervention for a client experiencing MI?
a- Administer morphine
b- Administer oxygen
c- Administer sublingual nitroglycerin
d- Obtain an ECG
95
323-A patient complains of stable angina presented with chest pain. The
physician ordered itroglycerine tablet sublingual. The nurse knows that the
right dose to give the patient is:
abcd-
0.1 mg
0.2 mg
0.4 mg
0.8 mg
324-Cancer patient during the shift and he surrounded by visitor and
laughing loudly before the shift end patient call the nurse and asked for
pain medication and said that his level of pain 8/10 (sever pain) patient
appears not on pain, what is the best intervention for this situation:
a- Give the patient extra dose of his pain medication
b- Ask the patient about the last time he had his medication
c- Ignore the patient as you are going to leave
d- Check the patient's file for history of drug abuse
325-A patient with pain was prescribed Ibuprofen 300 mg q 4 hours PRN
and morphine 5mg q 4 hours PRN. After 2 hours of receiving Ibuprofen the
patient still complaining of pain, what is the best intervention:
a- Wait 2 hours and give a second dose of ibuprofen
b- Give the patient his dose of morphine
c- Wait 1 hour and give the patient his morphine
d- Ask the patient to bear the pain
326- Which instruction should a nurse give the parents of an infant
undergoing cleft lip repair?
a- Offer the pacifier as needed
b- Lay the infant on his abdomen for sleep
c- Sit the infant up for each feeding
d- Loosen arm restraints every hour
96
327-Patient with abdominal incision, seven days postoperatively the patient
has pain at the site of incision which increased by walking and the incision
appear red and there's pus surrounding the incision site. This may indicate
which of the following:
a- Normal finding
b- Inflammation of the incision
c- Under nutrition
d- Good indicator
328- A preschooler is admitted to the hospital the day before scheduled
surgery. This is the child's first hospitalization. Which action will best help
reduce the child's anxiety about the upcoming surgery:
a- Begin preoperative teaching immediately
b- Describe preoperative and postoperative procedures in detail
c- Give the child dolls and medical equipment to play out the experience
d- Explain that the child will be put to sleep during surgery and won't feel
anything
329- Which teaching aid provided by the nurse is developmentally
appropriate for a preschooler who is about to have a bone marrow
puncture:
a- Dolls or puppets
b- Pamphlets or booklets
c- Colored diagrams
d- Commercial videotapes
330-A nurse is caring for an infant with spina bifida. Which assessment
findings suggest hydrocephalus?
a- Depressed fontanels and suture lines
b- Deep-set eyes, which appear to look upward only
c- Rapid increase in head size and irritability
97
d- Motor and sensory dysfunction in the foot and leg
331-Parents of a newborn with a unilateral cleft lip are concerned about
having the defect repaired. The nurse explains that a child with a cleft lip
usually undergoes surgical repair:
a- Immediately after birth
b- By 3 months of age
c- After 12 months of age
d- Varies in every case
332- The hepatitis B vaccine series should begin at what age:
a- Newborn
b- 2 months
c- 6 months
d- 12 months
333- A school age client admitted to the hospital because of decreased urine
output and per-orbital edema and diagnosed with glomerulonephritis.
Which of the following interventions would receive the highest priority:
a- Monitoring the vital signs every 4 hours
b- Monitoring intake and output every 12 hours
c- Monitoring the client weight daily
d- Monitoring serum electrolyte daily
334- The nurse is caring for a client with renal failure who is being treated
with peritoneal dialysis. Which assessment before and after peritoneal
dialysis is most valuable in evaluating the outcome of treatment:
a- Pulse rate
b- Body weight
c- Abdominal girth
98
d- Urine output
335- Which finding provides the best evidence that peritoneal dialysis is
achieving a therapeutic effect:
a- Urine output increases
b- Appetite improves
c- Potassium level falls
d- Red blood cell count is lower
336- When the nurse is advising the client about the potential complications
associated with peritoneal dialysis, which complication is most important to
include:
a- Pulmonary edema
b- Abdominal peritonitis
c- Abdominal hernia
d- Ruptured aorta
337- A client receiving hemodialysis treatment arrives at the hospital. He
complains of shortness of breath, and pedal edema is noted. His last
hemodialysis treatment was yesterday.
Blood pressure
200/100
Heart rate
110/min
Respiratory rate
36/m
Oxygen saturation
890/0 on room air
Which of the following interventions should be done first?
a- Administer oxygen
b- Elevate the foot of the bed
c- Restrict the client's fluids
d- Prepare the client for hemodialysis
99
338- Changes in personality and judgment are often associated with a lesion
in which of the following:
a- Frontal lobe
b- Parietal lobe
c- Broca's area
d- Wernicke's area
339- Anterior lobe of the brain is responsible for which of the following:
a- Personality
b- Movement
c- Speech
d- Memory
340- A history of smoking, abnormal permanent enlargement of the alveoli,
cough, and dyspnea suggest:
a- Asthma
b- Emphysema
c- Chronic bronchitis
d- Obstructive sleep apnea
341- Which of the following is the end of the first stage of labor:
a- Cervix dilated to 10 cm
b- Crowning of the presenting part
c- Increased bloody show
d- Contractions lasting up to 60 seconds
100
342- During the latent phase of the first stage of labor, how often should the
nurse plan to assess the fetal heart rate?
a- Every 5 to 15 minutes
b- Every 15 to 30 minutes
c- Every 30 to 60 minutes
d- Every 60 to 90 minutes
343- The nurse explains that oxytocin (pitocin) is given after delivery of the
baby and placenta for which purpose:
a- To increase the blood pressure
b- To prevent the uterus from inverting
c- To decrease the likelihood of hemorrhage
d- To prevent rupture of the uterus
344-A nurse in the delivery room is assisting with the delivery of a newborn
infant. After the delivery of the newborn, the nurse assists in delivering the
placenta. Which observation would indicate that the placenta has separated
from the uterine wall and is ready for delivery:
a- The umbilical cord shortens in length and changes in color
b- A soft and boggy uterus
c- Maternal complaints of severe uterine cramping
d- A sudden gush of dark blood from the introitus
345- Which vitamin deficiency increase the risk of neural tube defects:
a- Folic acid
b- Vitamin C
c- Vitamin B12
d- Calcium
101
346-A client has been diagnosed with folic acid deficiency. The nurse's
discharge teaching will focus on foods high in folic acid. Which of the
following foods has the highest folic acid level:
a- Citrus fruits
b- Raisins
c- Brewer's yeast
d- Eggs
347- Which of the following is the recommended daily dose of folic acid
during pregnancy:
a- 200 meg
b- 400 mcg
c- 800 meg
d- 1000 meg
348- Lochia normally disappears after how many days postpartum:
a- 5 days
b- 7-10 days
c- 18-21 days
d- 28-30 days
349- If the maintenance requirement of fluid for a child is 900 mil day, each
daily feeding account 120 ml, what is the number of feeding is required to
achieve this maintenance:
a- 5 feedings
b- 6 feedings
c- 7 feedings
d- 8 feedings
102
350- A 12 year- old patient had a cast removed from the left leg after
wearing if for eight weeks. The patient wants to resume sports as soon as
possible. In order to regain muscle strength lost while wearing cast, the
nurse will instruct the patient in performance of:
a- Resistive range of motion exercises to left leg
b- Passive range of motion exercises to right leg
c- Active- assistive range of motion exercises to the right leg
d- Active range of motion exercises to both legs
351- A 31 years- old woman with diabetes type 1 presents to the clinic with
fatigue, blurred vision, and loss of appetite. Her breath smells like fruit and
she leaves the room twice during the examination to use the toilet. She has
brought a little bottle of water with her that she finishes while at the clinic.
She reports that she has had a cold for the past three days, but has not
taken additional insulin during the illness
Blood pressure
130/70 mmhg
Heart rate
90/min
Respiratory rate
20/min
Body temperature
38.0 Coral
What is the most appropriate nursing diagnosis
a- Risk for impaired skin integrity related to circulation
b- Deficient knowledge related to illness management
c- Risk for fluid volume excess related to fluid intake
d- Imbalanced nutrition related to decreased appetite
352- Adult patient suffers from hoarseness of voice, and difficulty in speech
may be suffering from an injury of which of the following cranial nerves:
a- IX
c- XI
b- X
d- XII
103
353- The parent of a child with chronic asthma is hesitant to discipline
because the child often doesn't feel well. The nurse should encourage the
parent to:
a- Set consistent behavior limits
b- Be more lenient during times of illness
c- Cherish the limited time the child has to live
d- Avoid upsetting the child with limit- setting
354- A 55 year- old woman presented with pain in the joints of the hands.
She feels generally tired and has had a low grade fever for the past week.
On examination, the joints of the fingers on both hands are symmetrically
swollen.
Blood pressure
Heart rate
Respiratory rate
Temperature
Oxygen saturation
120/70 mmhg
82/min
18/min
38.0 C
99% on room air
Which type of arthritis is most likely?
a- Rheumatoid
b- Septic
c- Psoriatic
d- Osteoarthritis
355- A patient presented to the office for a physical examination. The
patient is found to be healthy and fit but occasionally drinks alcohol and
has unprotected sex.
What is the BEST nursing diagnosis?
a- Health- seeking behavior
b- Knowledge deficit, high- risk behaviors
c- Low self- esteem
d- Altered thought process
104
356- A seven year- old child presented to the emergency room with a
fracture of the right arm.
What would be the long- term goal for this patient?
a- Patient verbalizes decreased pain level
b- Nurse notes neurovascular checks are within normal limits
c- Nurse notes decreased swelling of the right arm
d- Patient performs activities of daily living without difficulty
357- The nurse is caring for a woman whose husband beats her regularly.
Which is the most important long-term goal for this woman?
a- Provide a long-term support group
b- Help her feel like a survivor
c- Point out the ways she behaved
d- Be able to blame the abuser
358-ln what position should a dyspneic patient be placed?
a- Prone
b- Recumbent
c- Semi-fowler's
d- Trendelenburg
359-A nurse is admitting a two year- old child with an umbilical hernia.
Which of the following interventions does NOT meet the child's
developmental needs?
a- Allowing the child to make choices when possible
b- Providing rooming in the unlimited visitation
c- Attempting to continue rituals used at home
d- Maintaining strict bedrest
105
360- A six year- old boy is in the postoperative care unit following a
tonsillectomy. The nurse observes that his gag reflex has returned and
removes the artificial airway. The patient then begins to cry and tells the
nurse that his throat hurts badly.
What type of data is the patient providing the nurse?
a- Objective
b- Inferential
c- Comparative
d- Subjective
361-A first time mother of a three week- old breastfed baby brings the
infant to the clinic and complains that her child has been forcefully
vomiting after feeding. He was born 40 week's gestation. Weighing 3.6
kilograms. He is constantly hungry and irritable. Examination reveals a
swollen abdomen and a palpable mass in the middle upper right quadrant.
What is the most likely health problem?
a- Intussusception
b- Pyloric stenosis
c- Gastroeosophageal reflux
d- Diaphragmatic hernia
106
362-A home care nurse reviews the second follow- up laboratory test results
of a patient with iron deficiency anemia. The nurse compares the recent
laboratory results to the results taken 3 months ago. The patient has been
eating food rich in iron as prescribed by the nutritionist.
What can be inferred about the progress of the treatment by comparing the
two laboratory values?
Laboratory values three months prior
Test
Albumin
Glucose
HCT
Hb
Blood urea nitrogen
Creatinine
Current laboratory values
Test
Albumin
Glucose
HCT
Hb
Blood urea nitrogen
Creatinine
Result
35
5.3
0.32
121
6.1
60
Result
33
5.7
0.41
139
5.7
60
Normal value
34-56 g/L
3.5-6.5 mmol/L
0.41-0.50
120-140 g/L
2.8 to 8.9 mmol/L
58-145 µml/L
Normal value
34-56 g/L
3.5-6.5 mmol/L
0.41-0.50
120-140 g/L
2.8 to 8.9 mmol/L
58-145 µmol/L
a- Patient's uptake of iron by diet is sufficient, and no additional intervention
is necessary
b- Patient's uptake of iron by diet is sufficient, but additional intervention is
necessary
c- Patient's uptake of iron by diet is insufficient, and additional intervention
is necessary
d- Patient's uptake of iron by diet is insufficient, and the physician's
immediate action is necessary
107
363- Which assessment finding would the nurse expect in an infant
diagnosed with pyloric stenosis?
abcd-
Abdominal rigidity
Ribbon-like stools
Visible waves of peristalsis
Rectal prolapse
364-A six year-old child with type 1 diabetes has an uncontrolled blood
glucose level. The child has been given regular insulin with minimal change
in glucose level in the first 30 minutes.
Which type of insulin has an onset of 15 minutes and a duration of no more
than 4 hours?
a- Aspart (Novo log)
b- Lispro (Humalog)
c- Semilete
d- Neutral protamine hagedom (NPH)
Type of insulin
Aspart (Novolog)
Lipsro (Humalog)
regular
semilente
NPH
Lente
Onset (minutes)
15
10-30
30-60
30-60
120
120
Maximal activity (hours)
1-3
1-2
2-4
2-4
4-12
8-10
Duration (hours)
3-5
2-4
6-9
10-12
24
24
365-A patient with cerebrovascular accident, left-sided hemiplegia, and
aphasia has nursing diagnosis of risk for aspiration related to swallowing
difficulties with a short-term goal that the patient will not aspirate. The
patient has undergone insertion of a percutaneous gastric endoscopy and
has all nutrition and fluid administered through the tube. The nurse
should:
a- Continue the care plan as written
b- Discontinue the risk for aspiration diagnosis
c- Revise the goals and interventions for the diagnosis
108
d- Add a nursing diagnosis of ineffective health maintenance
366-A nurse is evaluating a patient 5 days after a right total hip
replacement.
Which of the following goals is appropriate for the patient?
a- Maintain abduction without dislocation
b- Rest with legs elevated while sitting
c- Tie shoes and put on undergarments without assistive devices
d- Perform scissor-like exercises daily
367 - A three year- old child was admitted to the postoperative care unit
following a heart transplant. The nurse administers cyclosporine by
intravenous infusion. Fifteen minutes later the child has difficulty
breathing, his skin feels cold and clammy and he appears restless. Which is
the most appropriate initial nursing action?
a- Ensure airway patency
b- Administer oxygen therapy
c- Discontinue intravenous infusion
d- Administer intramuscular epinephrine
368- A 40 year-old woman is a gravida 2, para 2 and is currently trying to
conceive. Her previous pregnancy resulted in the birth of a baby with cleft
lip and palate. The patient is anxious and concerned about future
pregnancies and the nurse provides genetic counseling and reassurance.
Which food would most effectively prevent recurrence?
a- Green vegetables and citrus fruit
b- Eggs, milk and dairy products
c- Wheat, corn, rice, oats and rye
d- Beef, chicken and yellow vegetables
109
369- A 50 year-old woman presented with poor balance and coordination.
She says that she has developed pain on the outer aspect of the thighs and
the inner side of the arch of the foot. The nurse wishes to examine the
integrity of the affected lumbar spinal nerve root and performs a deep
tendon reflex examination.
Which reflexes would most likely be diminished?
a- Planter
b- Brachioradialis
c- Patellar
d- Achilles tendon
370- When administrating an intramuscular injection to an infant, which of
the following sites is appropriate for the nurse to use?
a- Rectus femoris
b- Deltoid
c- Dorsogluteal
d- Ventrogluteal
371- A 66 years-old woman is admitted to the hospital with a history of
hypertension. She presented with breathing difficulties that worsen with
activity and while sleeping. She is generally weak and feels that her heart
misses beats and that it sometimes beats loudly. An electrocardiogram
shows atrial fibrillation, right ventricular hypertrophy and deviation
toward the right.
Which nursing intervention is most appropriate for this patient?
a- Provide a bedside commode (portable toilet)
b- Place in right side-lying position
c- Encourage family and friends to visit
110
d- Encourage independent hygienic activities
372- Oral iron supplements are prescribed for a 6-year-old child with iron
deficiency anemia. The nurse instructs the mother to administer the iron
with which of the following food item to enhance absorption of iron:
a- Milk
b- Water
c- Apple juice
d- Orange juice
373- 9 months old infant was brought to the hospital by his parents because
he is crying most of the time. The examination revealed that he is suffering
from otitis media. Which of the following instructions you would include in
the care plan you suggest to the parents:
a- Give antibiotic as prescribed
b- Try to calm down your baby
c- Wash the ears regularly
d- Ignore the child's pain
374-A child with pneumonia was prescribed penicillin injection. Before
giving the injection to the child the nurse performed skin allergy test which
showed that the child has penicillin allergy. Which of the following is the
next step the nurse should take:
a- Stop the treatment and inform the doctor
b- Administer penicillin intramuscular
c- Postpone the does until the child is well
d- Ignore the test result and give the injection
111
375- The following table represents the blood pressure value for a patient in
3 successive days. Regarding the information given in the tablet, this patient
is considered:
a- Normal blood pressure
b- Pre-hypertension
c- Hypertensive
d- Hypotensive
First day
Second day
120/80
122/87
Third day
133/88
376-A patient with pneumonia with excessive mucous in the left lung, which
of the following is the best position to facilitate drainage of the mucous from
the lower left lung:
a- On the right side with trendelenburg position
b- On the left side with elevation of HOB
c- On the right side with elevation of the HOB
d- On the left side with trendelenburg position
377-A patient complains of left eye redness and itching, the doctor told you
to put atropine eye drops for the patient to examine his eye. The nurse
should instill the eye drops into:
a- The left eye
b- The right eye
c- Both right and left eyes
d- Neither of the eyes
112
378- The district nurse visits a 30 year-old woman at home following the
delivery of her second child, a full term girl. Following the delivery of her
first child, she had developed a breast infection and stopped breastfeeding
because of the pain. She asks the nurse how she can best prevent it with this
infant.
What is the most appropriate response?
a- Provide feeding on demand
b- Apply vitamin E cream to the nipples
c- Request a prophylactic antibiotic
d- Apply heat to the breasts after feeding
379- In planning home care for an immunocompromised child, the nurse
instructs the parents to use cream or emollients to prevent or manage dry
and cracked skin. A parent will BEST demonstrate understanding of the
rationale for this be stating:
a- Creams will prevent breaks in the skin and decrease the chance of
infection
b- Pleasantly scented creams will mask other less pleasant smells
c- Micronutrients in the creams will help prevent malnutrition
d- Creams will help prevent dehydration when my child does not drink
enough
380- The nurse is teaching a patient about spironolactone (Aldactone).
Which of the following instructions should review with the patient?
a- Increasing the intake of foods that are high in potassium
b- Taking the medication right before going to sleep
c- Avoid seasonings that are labeled as salt substitutes
d- Scheduling the medication so that a multivitamin is taken an hour later
113
381- A 49 year-old man with a diagnosis of alcoholic cirrhosis and ascites is
discharged from the hospital with a priority nursing diagnosis of altered
nutrition, less than body requirements.
Which dietary plan is most appropriate?
a- Low-protein, low sodium
b- Low sodium, high- protein
c- High- protein, low potassium
d- Low potassium, high protein
382-A patient who is 4 days postoperatively after a total hip replacement
surgery, is obese and has not been able to ambulate since the surgery. The
patient is now diaphoretic, has chills, and complains of pain in the thigh.
The MOST likely cause is:
a- Wound infection
b- Deep vein thrombosis (DVT)
c- Pulmonary edema
d- Dehydration
383- Which statement by the patient with hyperlipidemia shows a basic
understanding of the disease and its treatment?
a- Exercises has no effect on cholesterol levels
b- Hyperlipidemia is usually asymptomatic until significant target organ
damage is done
c- HDL cholesterol level of greater than 60mg/dL increases the chance of
coronary artery disease
d- Cholestyramine (questran) should be taken in the morning with other
medications
114
384- Which of the following describe the correct sequence of nursing
process:
a- Assessment, planning, diagnosis, implementation and evaluation
b- Planning, assessment, diagnosis, implementation and evaluation
c- Planning, diagnosis, implementation, assessment and evaluation
d- Assessment, implementation, evaluation, planning and diagnosis
385-A 14 year-old boy presented to the emergency department. He
complains of having progressively worsening stomach pain for the past
eight hours. On assessment, the pain is localized in the lower right
quadrant. He rates it as a level ten on the one-ten pain scale. Abdominal
palpation shows rebound tenderness in the lower right quadrant, and
positive McBurney and psoas signs.
Blood pressure
134-78 mmhg
Heart rate
88/min
Respiratory rate
24/min
Temperature
38.2 C
Test
result
normal values
WBC
12.4
4-10.5 X 109
Which intervention would be most appropriate to alleviate this patient's
pain?
a- Apply warm packs to affected area
b- Maintain semi-fowler's with legs up
c- Withhold solids and liquids
d- Administer analgesics
115
386-Female patient with a diagnosis of atrial fibrillation, the physician
ordered you to give her digoxin IV, but the patient refused to be injected
and told that she feel she is going to vomit, what you should do:
a- Change the IV digoxin to oral form
b- Force the patient to accept the medication
c- Inform the doctor about the situation
d- Don't give any medications to this patient
387-40 years old client complaining from abdominal pain, laboratory result
for stool analysis showed that there's occult blood in the stool. Which of the
following procedure would help for diagnosis of this patient:
a- Abdominal ultrasound
b- Abdominal x ray
c- Colonoscopy
d- MRI
388-A 7-year-old client is brought to the E.R. He's tachypneic and afebrile
and has a respiratory rate of 36 breaths/minute and a nonproductive cough.
He recently had a cold. From his history, the client may have which of the
following:
a- Acute asthma
b- Bronchial pneumonia
c- Chronic obstructive pulmonary disease (COPD)
d- Emphysema
389- Which of the following is a priority goal for the client with COPD:
a- Maintaining functional ability
b- Minimizing chest pain
c- Increasing carbon dioxide levels in the blood
d- Treating infectious agents
116
390- A 34-year-old woman with a history of asthma is admitted to the
emergency department. The nurse notes that the client is dyspneic, with a
respiratory rate of 35 breaths/minute, nasal flaring, and use of accessory
muscles. Auscultation of the lung fields reveals greatly diminished breath
sounds. Based on these findings, what action should the nurse take to
initiate care of the client:
a- Initiate oxygen therapy and reassess the client in 10 minutes.
b- Draw blood for an ABG analysis and send the client for a chest x-ray.
c- Encourage the client to relax and breathe slowly through the mouth
d- Administer bronchodilators
391- Which of the following is the best breathing pattern you should teach a
client with COPD:
a- Pursed-lip breathing
b- Deep breathing
c- Abdominal breathing
d- Slow lite breathing
392-A nurse instructs a female client to use the pursed-lip method of
breathing and the client asks the nurse about the purpose of this type of
breathing. The nurse responds, knowing that the primary purpose of
pursed-lip breathing is to:
a- Promote oxygen intake
b- Strengthen the diaphragm
c- Strengthen the intercostal muscles
d- Promote carbon dioxide elimination
117
393-An infant with tetralogy of Fallot is experiencing an attack involving
cyanosis and dyspnea. Which position should the infant be placed in:
a- Fowler's
b- Knee-chest
c- Trcndelcnburg's
d- Prone
394-A client is unresponsive and has been brought to the emergency
department.
Initial laboratory results reveal:
Serum K +
3.6 mmol/L
Glucose
26 mg/dl
Hemoglobin
12. 6 gm/dl
Carbon dioxide
26. 2 mmol/L. The nurse will anticipate:
a- Dextrose by mouth
b- 50% dextrose IV
c- 1 unit of packed cells
d- 10 mEq of KCl over 0.5 hour
395- When a client has a nephrostomy tube, the priority nursing care is to:
a- Ensure drainage of urine
b- "Milk" the tube every 2 hours
c- Keep an accurate record of intake and output
d- Instill 2 ml of normal saline solution every shift
118
396- Which of the following is the most common symptom of myocardial
infarction (MI):
a- Chest pain
b- Dyspnea
c- Edema
d- Palpitations
397- A client has surgery for a perforated appendix with localized
peritonitis. In which position should the nurse place the client:
a- Sims position
b- Trendelenburg
c- Semi-fowlers
d- Dorsal recumbent
398- Which of the following position should the client with appendicitis
assume to relieve pain:
a- Prone
b- Sitting
c- Supine
d- Lying with legs drawn up
399-A women breast feed her infant for one or two hours and her infant
cries most of the time and she feels pain in her breast, which of the
following instructions are appropriate for the nurse to give the mother:
a- Regulate breast feeding every 3 hours
b- That's normal feeding problem
c- Shift to bottle feeding
d- Start weaning your baby
119
400- While caring for a child with a ventriculoperitoneal shunt revision, the
nurse finds the patient lying with the head and feet flexed back. The nurse
should call for help and prepare for a (n):
a- Spinal tap
b- Shunt culture
c- Electrocardiogram
d- Ventricular tap
401- While assessing a child with pulmonary stenosis, the nurse should give
PRIORITY to:
a- Deep tendon reflexes
b- Urinary output
c- Exercise tolerance
d- Pattern of food intake
402- A 25 years-old male patient suffered a spinal cord injury at the T -4
level and is being cared for in hospital. The nurse enters the patient's room
and finds the patient sitting upright and looking anxious and restless. He
complains of sudden headache and nausea. Sweat forms on his forehead yet
his feet are cool to touch
Blood pressure
150/100 mmhg
Heart rate
55/min
Respiratory rate
28/min
Temperature
37.1C
What nursing intervention is initially most appropriate?
a- Assess for a full bladder
b- Lower the head of the bed 30 degrees
c- Loosen clothing and bed sheets
d- Apply heating pad to lower extremities
120
403- An 80 year-old man presents to the hospital with chronic fatigue,
dyspepsia and constipation. On examination he is jaundiced, has red palms,
dilated veins around the umbilicus, the abdomen is very distended and he
has black, tarry stool on a rectal exam. He is noted to be lethargic and have
a flat tone. A paracentesis reveals clear colored fluid with low protein
content.
What is most likely diagnosis?
a- Gilbert's syndrome
b- Thalassemia
c- Sickle cell crisis
d- Cirrhosis
404Admission
Report at 22:4500:45 .m.
Diagnosis
14 years old Remains febrile, maximum temperature this Bacterial
admitted
shift 39.3C
pneumorua
18 hours ago
Respiration rate 12-14 per minute Heart rate
100-180 per minute Blood pressure level
within baseline normal range
Next dose of intravenous anti-infective is due
at 08:00 (8:00 a.m.)
Refer to the accompanying figure.
The nurse should plan to monitor which of the following at least every 4
hours?
a- Vital signs
b- Urinary output
c- Food intake
121
d- Level of activity
405- Which instruction take priority in reducing anxiety related to surgery?
a- Surgical procedure and postoperative exercises
b- Risk of infection after surgery
c- Advanced directive and what it means
d- Pre-operative laboratory result and what to expect on it
406- When caring for a patient with an ostomy, the nurse knows that extra
protection for peristomal skin is MOST important for those patients with a
(an):
a- Ileostomy
b- Ascending colostomy
c- Transverse colostomy
d- Sigmoid colostomy
407- Which of the following nursing diagnosis takes PRIORITY for a
patient admitted to the critical care unit with diabetic ketoacidosis?
a- Deficient fluid volume secondary to hyperglycemia
b- Risk of infection secondary to weakened immune system
c- Deficient knowledge of cause and prevention
d- Imbalanced nutrition related to hyperglycemic state
122
408-A 69 year-old man is admitted to the intensive care unit following
cardiac surgery. Two hours after admission, the nurse performs a routine
assessment and notes the patient's chest tube drainage is 200 milliliters and
a dark red color. He has had 60 milliliters output from the indwelling
urinary catheter
Blood pressure
138/68 mmhg
Heart rate
76/min
Respiratory rate
16/min
Body temperature
Oxygen saturation
37.0C oral
94% 6L/min nasal cannula
Which finding should be reported to the doctor?
a- Dark red chest tube drainage
b- Urinary output
c- Oxygen saturation
d- Chest tube output volume
409- To minimize a toddler from scratching and picking at healing skin
graft, the nurse should utilize:
a- Mild sedatives
b- Hand mittens
c- Punishment for picking
d- Distractions
123
410- The following pain medications are ordered for a patient who had a
right leg amputation: Oxycodone 5 mg every 4 hours as needed and
morphine 5 mg every 4 hours as needed. The nurse administered oxycodone
2 hours ago, but the patient reports pain rated 8 on a scale of 0 (no pain) to
10 (severe pain) as the dressing change begins
After evaluating the effectiveness of the pain medication, what action
should the nurse take?
Blood pressure
Heart rate
Respiratory rate
Temperature
169/98 mmhg
112/min
22/min
36.7C
a- Administer additional oxycodone 5 mg
b- Administer morphine 5 mg
c- Change the dressing quickly
d- Ask the patient to wait 2 hours
411-A 55 year-old man presented to the clinic with complaints of numbness
and tingling in his feet for the past 3 months. He appears thin and his skin
is pale. Examination confirms the loss of vibration sense. He also has
altered proprioception. He had a subtotal gastrectomy two years ago (see
lab results)
Test
MCH
HCT
0.31
MCV
Reticulocyte count
result
2.72
normal values
1.45-2.01 fmol/cell
0.41-0.50
81-97 fl
0.5%-2.50/0
105
1.5
Which supplement would most likely improve his condition?
a- Iron
b- B12
c- Folate
d- Calcium
124
412-A 35 year-old man is hospitalized following a blunt chest injury. He has
chest pain, breathing difficulty and asymmetrical lung expansion. Chest
auscultation reveals decreased lung sounds on the left side. The jugular
veins are distended and there is tracheal deviation to the right.
Blood pressure
92/54 mmhg
Heart rate
120/min
Respiratory rate 24/min
Temperature
36.2C
SaO2
88% on oxygen
Which initial intervention is most appropriate?
a- Send patient for X-ray examination
b- Administer bronchodilators
c- Administer intravenous fluid bolus
d- Prepare for needle decompression
413-A patient is being followed in the clinic for hypertension, adult onset
diabetes, and obesity. The patient is apathetic about learning about
nutritional guidelines to reach the goals of weight loss and consumption of a
healthy diet. The patient admitted to eating "whatever is put in front of
me"
Which of the following actions would the nurse take?
a- Collaborate with the patient to set goals
b- Add a nursing diagnosis of non-compliance
c- Refer for psychiatric screening for depression
d- Discuss nutritional interventions with the spouse
125
414- The nurse is measuring the chest tube drainage of a patient who had
open heart surgery 4 hours ago.
Which of the following is the MAXIMUM hourly amount of chest tube
drainage is expected in this timeframe?
a- 100 ml
b- 200 ml
c- 300 ml
d- 400 ml
415-A nurse is caring for a three year-old child with a fractured arm.
Which of the following interventions is the MOST appropriate for pain
management?
a- Administer analgesics when necessary
b- Assess pain once a shift
c- Anticipate pain and intervene early
d- Encourage the use of self-quieting techniques
416-After a hearing restoration operation, a patient has no signs of
complications and soon recovers.
Which of the following is an expected outcome 5 days after the hearing
restoration surgery?
a- Regain full hearing
b- Minimal facial nerve paralysis
c- Minimal urinary incontinence
d- Ambulates without difficulty
126
417A
B
C
D
E
F
G
Goals for the next three months, the patient with:
Not have any migraine headaches
Decrease the number of migraine headache
Learn to tolerate her migraine headaches
Continue to take prescribed preventative medications daily
Initiate complementaty pain management strategies as needed
Wean self from all pain medications
Keep a pain diary and recognize triggers and auras before migraine
headache commence
An adolescent patient with a history of migraine headaches for the past
year has been followed for pain management. The headaches have
decreased in severity and now occur only occasionally
What are the BEST three long term goals?
a- A, D,F
b- B, D, F
c- C, E, F
d- D, E, G
418-A community health nurse visits a patient who had a cerebrovascular
accident. The patient is at risk for deficient fluid volume due to voluntary
reduction of fluid intake to avoid the use of the bathroom. The nurse
educates the patient on the importance of drinking fluids and maintaining
hydration/ Which of the following indicates the efficacy of the nursing
intervention?
a- Amber color urine
b- Respiration of 35
c- Tacycardia
d- Moist mucous membrane
127
419-A patient with diabetes mellitus and multiple sclerosis has been
prescribed baclofen (Lioresal). The nurse knows this medication may result
in?
a- Increased insulin needs
b- Renal failure
c- Optic neuritis
d- Muscle tremors
420-
A
B
C
D
E
F
Nursing diagnosis
Impaired physical mobility
Activity intolerance
Alternation of comfort
Risk of alternation in skin integrity
Knowledge deficit
Decreased cardiac cutout
A patient with Alzheimer's disease and severe cardiomyopathy presents to
the hospital with a fractured left hip. The patient is on bed rest until a
cardiologist clears the patient to have surgery.
Which nursing diagnosis have the highest priority?
a- E and F
b- D and C
c- C and F
d- D and F
128
421-A 19 year-old woman telephones the nurse and complains of difficulty
of breathing. The symptoms developed while exercising that morning. She
has a productive cough with thick mucus secretions and wheezing. The
nurse hears that the breathing problems do not interfere with talking on the
phone. The woman plans to attend the clinic but must wait for two hours
before she can be driven there.
Which treatment should be recommended first by the physician?
a- Purse-lip breathing
b- Deep breathing and coughing
c- Decrease physical activity
d- B2 agonist administration
422- A patient is recovering from surgery using spinal anesthesia. The
patient developed a spinal headache.
Which of the following nursing actions would be MOST appropriate?
a- Elevate the head of the bed 30 degrees
b- Keep the patient well hydrated
c- Limit intake of salty foods
d- Lower the temperature of the room
423-A patient is preparing for a total knee replacement. During the
preoperative interview process the patient reports an allergic reaction to
penicillin. Which of the following is considered a side effect and not a true
allergy to medication?
a- Shortness of breath
b- Tingling lips and tongue
c- Rash
d- Upset stomach
129
424-Prior to providing care for a hospitalized infant, the nurse who focuses
on preventive measures must:
a- Introduce self to parents
b- Perform hand hygiene
c- Have a witness present
d- Assess the child's developmental level
425- As identified by Dr. Elizabeth Kubler-Ross, which stage of dying is
characterized by the transition from, "NO, not me" to "Yes, me, but ... "
a- Anger
b- Depression
c- Acceptance
d- Bargaining
426- A 50 year-old male presents to the medical office 3 weeks after cardiac
surgery with complaints of a feeling of weakness, difficulty breathing, and
joint pains. Upon examination the nurse finds a fever and a friction rub on
auscultation of the chest. The nurse recognizes that the MOST likely
surgical complication is:
a- Neuropsychological dysfunction
b- Postpericardiotomy syndrome
c- Cardiac tamponade
d- Phrenic nerve damage
130
427-A patient with heart failure has the following vital signs, which of these
vital signs should be reported to the physician prior to administrating the
next dose of digoxin?
Blood pressure
136/84 mmhg
Heart rate
48/min
Respiratory rate
20/min
Temperature
37.1C
a- Blood pressure
b- Pulse
c- Temperature
d- Respiratory rate
428- The floating ribs that are not attached with sternum are:
a- Ribs 7&8
b- Ribs 9&10
c- Ribs 11&12
d- Ribs 9&10&11&12
429-A pediatric nurse is providing medication instructions to the mother of
a 13 year-old boy who is starting anti-depressant therapy. The mother
appears unfocused, agitated and confused and asks the nurse to repeat the
instructions several times. She tells the nurse that she is concerned that she
might administer the medication incorrectly. The nurse is concerned that
the mother is not following the instructions.
Which intervention would be most appropriate?
a- Reassure her that doubts are normal
b- Reinforce the importance of correct dosage
c- Refocus the teaching with printed material
d- Take a small break and the continue
131
430-A child is treated for superficial (first-degree) thermal burns to the
thigh. The child is in great discomfort and does not eat.
Which of the following diagnosis should receive PRIORITY?
a- Altered nutrition
b- Impaired skin integrity
c- Risk of infection
d- Acute pain
431-A 3-week-old infant is hospitalized with jaundice. When considering
the fluid needs relative to body size of the infant as compared to the fluid
needs of an adult. The nurse knows this infant requires:
a- Less fluids
b- More fluids
c- Same amount of fluids
d- Much less fluids
432- Which of the following statement is most accurate?
a- Girls have more ADHD than boys
b- Boys are more frequently diagnosed with chronic illnesses
c- Boys are less frequently diagnosed with chronic illnesses
d- Girls have more other chronic illness than ADHD
132
433- What is the percentage of boy with attention deficit hyperactivity
disorder?
a- 12%
b- 16%
c- 8%
d- 4%
434- The nurse is observing a detailed neurological assessment on a client
with a suspected brain tumor. When performing the Romberg test, the
client sways when the eyes are both open and closed. What does this
indicate?
a- The problem is probably in the cerebellum
b- It is a position sense abnormality
c- This is not an abnormal test result
d- The client has lost proprioception
435-An 18 year-old woman who broke her right ankle is seen in the
physician's office one week after the cast was removed.
Which of the following is the short term goal for this patient?
a- Walk 100 feet with crutches
b- Walk completely independent
c- Relieve the pain
133
d- Strict bedrest
436-A client with multiple sclerosis has been prescribed the drug baclofen
(Lioresal). What is the action of this drug?
a- Reduces spasticity
b- Skeletal muscle relaxation
c- Immune suppression
d- Prevents viral infections
437- What is the average incubation period of Hepatitis A?
a- 30 days
b- 60 days
c- 50 days
d- 14 days
438-After the health care team meets to discuss the client's nursing needs,
the nursing diagnosis "Disturbed body image" is added to the care plan.
The best rationale for adding this nursing diagnosis to the care plan in the
case of a female is that females with Cushing's syndrome typically
experience which physiologic effect?
a- Masculine characteristics
b- Heavy menstrual flow
c- Extreme weight loss
d- Large, pendulous breasts
134
439- A 64-year-old client with uterine cancer is scheduled to undergo an
abdominal hysterectomy under general anesthesia. Before the client returns
from the postanesthesia care unit, the registered nurse asks the licensed
practical nurse to help revise the care plan for the client who has undergone
a hysterectomy. Which nursing diagnosis is most appropriate for the nurse
to add to the client's care plan at this time?
a- Risk for ineffective airway clearance
b- Risk for imbalanced nutrition
c- Ineffective coping
d- Impaired verbal communication
440- The nurse initiates a teaching plan for the client with Parkinson's
disease.
Which instruction should be the nurse's priority in this situation?
a- Steps to enhance the client's immune system
b- Importance of maintaining a balanced diet
c- Need to remove all safety hazards
d- Importance of social interactions
441- The charge nurse enters the nursing diagnosis "Risk for ineffective
airway clearance related to an inability to swallow" on the client's care
plan. Which nursing intervention is most appropriate for managing the
identified problem?
a- Keeping the client supine
b- Removing all head pillows
c- Performing oral suctioning
d- Providing frequent oral hygiene
135
442- The client has returned from surgery with a leg cast, and the nurse is
assisting the client back to bed. Which of the following would the nurse
identify as the highest priority when documenting the postoperative
circulation status of the recently casted extremity?
a- Adequate neurovascular functioning
b- Minimal pain on movement
c- Vital signs within normal limits
d- No drainage noted on the case
443-After X-rays are taken of the head, neck, and spine, the client is
diagnosed with a head injury and admitted for inpatient care. When
assessing the client with a head injury, which of the following should receive
priority attention?
a- Lung sounds
b- Clarity of speech
c- Mobility of fingers
d- Pupillary responses
444- The client experiencing a severe allergic reaction becomes pulseless.
The nurse shakes the client, shout the client's name but gets no response,
and activates the emergency medical response system. Which nursing
action becomes the next priority?
a- Administer a single blow to the sternum
b- Give two quick breaths that make the chest visibly rise
c- Begin chest compressions at a rate of 100 per minute
d- Administer an epinephrine (Adrenalin) injection
136
445-A 50-year-old client is scheduled for a heart transplant tomorrow. The
night nurse is asked to review the surgical procedure with the client.
Because of the client's anxiety, the client has difficulty comprehending the
nurse's information. During the postoperative period, what is the best
rationale for the nurse frequently assessing the client's fluid status?
a- Urine retention is common after a heart transplant
b- Urine output is an indication of perfusion to the kidneys
c- Hydration determines when the client needs to be transfused
d- Hydration indicates when fluids should be increased
446-A nurse is assigned to care for a patient with a diagnosis of thrombotic
stroke. The nurse knows that this type of stroke is MOST LIKELY caused
by:
a- Blockage of large vessels as a result of atherosclerosis
b- Emboli produced from valvular heart disease
c- Decreased cerebral blood flow due to circulatory failure
d- A temporary disruption in oxygenation of the brain
447-A nurse obtains a urine dipstick analysis sample from a 35 year-old
woman who reports having burning sensation with urination and a sense of
urgency and frequency. She had been diagnosed with the condition six
months previously and was prescribed a course of antibiotics.
Urinal sis
Colour
Odour
Appearance
Leukocyte esterase
Nitrites
Results
Dark yellow
Abnormal
Turbid
Positive
Positive
Normal values
Straw-coloured
Almost nothing
Clear
Negative
Negative
Which type of pharmacological treatment is most likely?
a- Anti-viral
b- Anti-fungal
c- Anti-bacterial
d- Anti-parasitic
137
448- A 30 year-old woman has been prescribe albuterol PRN and
prophylactic inhaled corticosteroids to be taken once per day. She has been
taking the prophylactic as prescribed but has needed to use albuterol more
often than usually. She has a chronic cough and often has air hunger.
Which intervention is initially most appropriate?
a- Refer for a chest X-ray
b- Administer magnesium sulfate
c- Assess peak flow measure
d- Perform arterial blood gas sampling
449- Which of the following drugs is not used in the treatment of
pulmonary embolism?
a- Heparin
b- Warfarin
c- Digoxin
d- Streptokinase
450-A home care patient with chronic obstructive pulmonary disease
(COPD) reports an upset stomach. The patient is taking theophylline
(Theo-Dur) and triamcinolone acetonide (Azmacort), The nurse should
instruct the patient to take:
a- Theo- Dur on an empty stomach
b- Theo- Dur and Azmacort at same time
c- Theo- Dur and Azmacort 12 hours apart
d- Theo-Dur with milk or crackers
138
451-A 65 year-old woman presented to her care provider with complaints of
bright red blood in the stool, a loss of appetite, a feeling of fullness and
fatigue. She had lost 5 kilograms in the past three weeks without dieting. A
faecal occult blood test is positive and the patient is scheduled for additional
screening test. Which screening test is most likely for this patient?
a- Barium enema
b- Colonoscopy
c- Endoscopy
d- Computed tomography scan
452-A baby girl was born prematurely at 33 weeks gestation due to placenta
abruption. She is now two months old and has a potential hearing deficit. In
the neonatal intensive care unit (NICU) she had passed the initial screening
tests for hearing but has significant risk factors that suggest further
evaluation.
Which autonomic reflex should be further evaluated?
a- Moro
b- Babinski
c- Pupillary
d- Tonic neck
453-A 45 year-old woman presented with a generalized rash that is not
itchy. She reports that she has had the problem for the past 15 years.
Examination reveals a well-outlined, reddish plaque over the right gluteal
fold. The plaque has scales over it and is cracked in some areas.
Which intervention is initially appropriate?
a- Apply topical cream to the affected area
b- Expose area to sunlight for twenty minutes daily
c- Maintain immunosuppressant therapy regimen
d- Increase dietary intake of vitamin A
139
454- The patient is receiving mechanical ventilation set at fraction of
inspired oxygen (FiO2) 100%.
The nurse should understand that which of the following can improve this
patient's oxygenation?
a- Adding positive end expiratory pressure (PEEP)
b- Place the patient in trendelenburg position
c- Increasing the FiO2
d- Suctioning the patient hourly
455-A nurse is caring for a 3-week-old infant who just admitted to the
hospital.
Which of the following nursing interventions does NOT support this
infant's basic emotional and social needs?
a- Provide for continual contact between parents and infant
b- Actively involve parents in caring for the infant
c- Keep the infant's environment quite, dim and free of sensory stimulation
d- Foster infant-sibling relationships as appropriate
456-A community health nurse is teaching a health class about infectious
diseases process.
The nurse instructs the class that rabies would be considered which of the
following type of infection?
a- Viral
b- Protozoan
c- Fungal
d- Bacterial
140
457-A couple who wants to conceive but has been unsuccessful during the
last 2 years has undergone many diagnostic procedures. When discussing
the situation with the nurse, one partner states, "We know several friends
in our age group and all of them have their own child already, why can't we
have one?" Which of the following would be the most pertinent nursing
diagnosis for this couple?
a- Fear related to the unknown
b- Pain related to numerous procedures
c- Ineffective family coping related to infertility
d- Self-esteem disturbance related to infertility
458- A 64 year old male client with a long history of cardiovascular
problem including hypertension and angina is to be scheduled for cardiac
catheterization. During pre-cardiac catheterization teaching, the nurse
should inform the client that the primary purpose of the procedure is:
a- To determine the existence of CHD
b- To visualize the disease process in the coronary arteries
c- To obtain the heart chambers pressure
d- To measure oxygen content of different heart chambers
459- A gravida 3 para 2 is admitted to the labor unit. Vaginal exam reveals
that the client's cervix is 4cm dilated. The patient complains of pain which
she stated to be 7/l0.at this time and until a full cervix dilatation is achieved,
what is the priority nursing goal for such patient at this time is:
a- Pain management
b- Prevent fetal distress
c- Preparing the patient for anesthesia
d- Keeping this patient NPO
141
460-A client with iron deficiency anemia is scheduled for discharge. Which
instruction about prescribed ferrous gluconate therapy should the nurse
include in the teaching plan?
a- "Take the medication with an antacid."
b- "Take the medication with a glass of milk."
c- "Take the medication with cereal."
d- "Take the medication on an empty stomach."
46l-After 4 hours of active labor, the nurse notes that the contractions of a
primigravida client are not strong enough to dilate the cervix. Which of the
following would the nurse anticipate doing?
a- Obtaining an order to begin IV oxytocin infusion
b- Administering a light sedative to allow the patient to rest for several hour
c- Preparing for a cesarean section for failure to progress
d- Increasing the encouragement to the patient when pushing begins
462-A 24 year-old woman was prescribed loratidine (Claritin) 10 mg tablet
q 12 hours a.c. for allergy.
The nurse reviews the medication order and explains to the patient the
relation of this drug to meals is as following:
a- The drug is to be taken after meals
b- The drug is to be taken during meals
c- The drug is to be taken before meals
d- The drug is to be taken away from meals
142
463-An elderly client is experiencing an alteration in his equilibrium and
coordinated muscle movements. The nurse realizes that these functions are
controlled by which area of the nervous system?
a- Brain stem
b- Cerebrum
c- Diencephalon
d- Cerebellum
464-A 32-year-old female is admitted for a hemorrhoidectomy. During the
nursing assessment, all of the following factors are elicited. Which one is
most likely to have contributed to the development of hemorrhoids?
a- The client states that she usually cleans herself from back to front after a
bowel movement
b- The client says her mother and grandmother had hemorrhoids
c- The client has had four pregnancies
d- The client eats bran every day
465- A client who has hepatitis A asks, "How could I have gotten this
disease?" what is the nurse's best response?
a- You may have eaten contaminated food
b- You could have contracted the disease by using intravenous drugs
c- You must have received and infected blood
d- You could have contracted the disease by engaging in unprotected sex
466-A client undergoes right mastectomy for carcinoma. When teaching the
client post-mastectomy exercises, it is important for the nurse to:
a- Exercise both arms simultaneously
b- Exercise the right arm only
c- Have the client wear a sling between exercise periods
d- Wait until the incision has healed
143
467- The following picture represents a newborn reflex known as?
a- Rooting reflex
b- Moro reflex
c- Grasping reflex
d- Startle reflex
468- The nurse is caring for a client who has had a right modified radical
mastectomy this morning. Which exercise should the nurse encourage the
client to perform this evening?
a- Hair combing exercises with the right arm
b- Wall climbing exercises with the right arm
c- Movement of the fingers and wrists of the right arm
d- Exercises of the left arm only
469- A 38-year-old client who has mitral stenosis is hospitalized for a valve
replacement. Which condition is the client most likely to report having had
earlier in life?
a- Meningitis
b- Syphilis
c- Rheumatic fever
d- Rubella
144
470-An 82-year-old woman who has Alzheimer's disease is admitted to the
acute care unit. She frequently gets out of bed and wanders in the hall,
unable to find her way back to her room. She even gets in the beds of other
clients. What nursing action is most appropriate for this client?
a- Restrain her so she will not wander in the halls
b- Ask her roommate to call the nurse whenever she leaves the room
c- Punish her when she gets in a bed other than her own
d- Put her favorite picture on the door to her room
a- 471-A 35-year-old man is admitted with severe renal colic. The nurse
should monitor this man for possible complications. Which of the
following is a Oliguria
complication of renal colic?
b- Anemia
c- Polyuria
d- Hypertension
472-A nurse discusses high-risk complications with a group of women at a
prenatal clinic. Which client would the nurse identify as being at highest
risk for developing complications during pregnancy?
a- A 25-year-old gravida I client
b- A client with the placenta implanted on the fundus of the uterus
c- A client who has nausea and vomiting during the first trimester
d- A 30-year-old client with DM
145
473-A 56 year-old man was brought to the emergency room by his relatives,
on examination he appears sick and has severe weakness. One of his
relatives told the nurse that the man eats nearly nothing and have been
crying most of the time for 3 months now since he lost his son in an
accident. The nurse knows that this patient is suffering from:
a- Mania
b- Insomnia
c- Depression
d- Schizophrenia
474- Which of the following drugs is used for the treatment of digitalis
toxicity?
a- Protamine sulfate
b- Streptokinase
c- Theophylline
d- Digoxin immune Fab
475-A female client is admitted with a diagnosis of acute renal failure. She
is awake, alert, oriented, and complaining of severe back pain, nausea and
vomiting and abdominal cramps.
Blood-pressure
Pulse
110
Respiratory rate
Temperature
Sodium
Potassium
100/70 mm Hg
30
38°C oral
120 mEq/L
5.2mEq/L
Urinary output for the first 8 hours is 50 ml
The client is displaying signs of which electrolyte imbalance?
a- Hyponatremia
b- Hyperkalemia
c- Hyperphosphatemia
d- Hypercalcemia
146
Some important notes to remember
 Renal failure - acute: low protein, high carbohydrate, low sodium
(oliguric phase), high protein, high calorie, restricted fluid (diuretic phase)
 Renal failure - chronic: low protein, low sodium, low potassium
 Normal color of stoma (colonostomy): pink or red
 Teething starts at: 7 months and till 16 months
 After Laparoscopic cholecystectomy: patient regain normal activity
after 12-14 days
 After renal surgery the most common complications: DVT
 Typhoid: transmission (contact) - isolation (contact isolation)
 Food rich in vitamin C: orange, Broccoli, strawberries, tomato
 MRSA isolation: contact
 Complication of hemorrhoidectomy: infection and thrombosis
 HCV transmitted by: blood
 Test to assess anemia patient: ferritin and hemoglobin
 Psoriasis takes the shape of: scales
 Normal capillary refill: less than 3 seconds
 Child has limb swelling and ascites, the main cause of swelling:
Increased hydrostatic pressure
 Sign of malnutrition for child: open fontanel, low weight for his age
147